PMHN Practice Exam 3

अब Quizwiz के साथ अपने होमवर्क और परीक्षाओं को एस करें!

Which of the following feedback is specific and descriptive? "You were very sarcastic in the group meeting today." "Marvin became upset when you made a joke about his failure to maintain sobriety." "You tend to be thoughtless when you address other patients in the group." "You should treat others with more respect in group meetings."

"Marvin became upset when you made a joke about his failure to maintain sobriety." Rationale: The feedback that is specific and descriptive is "Marvin became upset when you made a joke about his failure to maintain sobriety" because it gives the essential facts. "You were very sarcastic in the group meeting today" is evaluative ("very sarcastic") without outlining the specific problem. "You tend to be thoughtless when you address other patient in the group" is too general. "You should treat others with more respect in group meetings" is giving advice ("you should") as opposed to feedback.

If a patient has a nursing diagnosis of social isolation, which of the descriptions is appropriate to document in the patient's electronic health record (EHR)? "Patient exhibiting social isolation and lack of cooperation with group activities." "Patient increasingly withdrawn and uncooperative with therapy." "Patient stays alone in room, stating 'Go away,' when asked to participate in group activities." "Patient isolating himself and being increasingly unfriendly and resistive to group activities."

"Patient stays alone in room, stating 'Go away,' when asked to participate in group activities." Rationale: If a patient has a diagnosis of social isolation, the description that is appropriate to document in the patient's electronic health record (EHR) is, "Patient stays alone in room, stating 'Go away,' when asked to participate in group activities." In documenting, it is important to describe rather than label because labels, such as "withdrawn," "isolating," and "uncooperative" may be interpreted in various ways and do not provide a clear picture of the patient's behavior.

Which of the following statements could be considered a violation of professional conduct? "You look very nice today." "Did you have a good weekend?" "I like your shoes. Are they comfortable?" "I think your comments about the patient's sexuality are inappropriate."

"You look very nice today." Rationale: "You look very nice today" could be considered a violation of professional conduct if the person to whom the comment is directed feels uncomfortable or if other people hear the comment and feel uncomfortable. Even though people working together often forge friendships, comments about physical appearance in the workplace are almost always inappropriate and can be easily misconstrued. Commenting on shoes is probably safe as are general questions, such as "Did you have a good weekend?" It is acceptable to directly address inappropriate comments by others.

Considering assertiveness training, an example of "clouding/fogging" in response to the statement, "You should be fired for the way you handled that situation," is: "You're right. I could have handled that situation better." "Of course you're right. We all know you are always right." "What exactly did I do wrong?" "I can see you're upset. Let's discuss this later."

"You're right. I could have handled that situation better." Rationale: Considering assertiveness training, an example of "clouding/fogging" in response to the statement, "You should be fired for the way you handled that situation," is "You're right. I could have handled that situation better." Clouding/fogging addresses only part of the issue (handling the situation better) and "clouds" or by-steps the other (should be fired) as a way to assert control of the issue. Other types of assertive responses include agreeing assertively, inquiring assertively, defusing, and responding with irony.

An example of primary prevention is: -Providing parenting classes for prospective parents -Referring patients for treatment -Conducting an ongoing assessment of high-risk patients -Monitoring the effectiveness of treatment and services

-Providing parenting classes for prospective parents Rationale: An example of primary prevention is providing parenting classes for prospective parents because the goal is to prevent issues, such as abuse and neglect, by providing education and support. Primary prevention goals are to identify high-risk populations and to intervene in order to decrease risk or to minimize negative consequences. Other examples of primary prevention include teaching mental health concepts to community members, providing education on dealing with life transitions (widowhood, marriage, adolescence, empty-nest), and educating people about the negative effects of alcohol and drugs.

A female patient with a methadone overdose is undergoing Q-T monitoring. The normal range for the Q- T interval is: 0.33 to 0.35 seconds 0.36 to 0.44 seconds 0.45 to 0.50 seconds 0.51 to 0.53 seconds

0.36 to 0.44 seconds Rationale: The normal range for the Q-T interval is 0.36 to 0.44 seconds with the normal for males <0.43 seconds and females <0.45 seconds. Critical measure is >0.50 seconds. The Q-T measure varies somewhat from person to person and is longer with a slow pulse than a fast pulse. When monitoring, the psychiatric and mental health nurse should monitor with a lead in which the T wave is clearly evident on the tracing. Q-T prolongation may result from numerous drugs, including amitriptyline, aripiprazole, chlorpromazine, citalopram, olanzapine, quetiapine, sertraline, and methadone.

For a patient taking lithium to control bipolar disorder, the serum level for maintenance should be: 0.5 to 1.5 mEq/L 1.5 to 2.0 mEq/L 2 to 3 mEq/L 3 to 4 mEq/L

0.5 to 1.5 mEq/L Rationale: The lithium level for maintenance should be 0.5 to 1.5 mEq/L. If the level increases to 1.5 to 2 mEq/L, then the next dose of medication should be withheld and the serum level of the drug tested. A further increase to 2 to 3 mEq/L is of considerable concern as the patient may exhibit moderate signs of toxicity. The patient requires immediate IV fluids as well as withholding of lithium. A level of 3 mEq/L or above is life threatening and requires emergent intervention, sometimes including dialysis to lower lithium levels.

In order for a patient with a terminal disease to be admitted to a hospice program under Medicare, the physician must certify that the patient's death is expected within: 1 month 6 months 8 months 12 months

6 months Rationale: In order for a patient with a terminal disease to be admitted to a hospice program under Medicare, the physician must certify that the patient's death is expected within 6 months. However, since the time of death is not always predictable, the patient can be certified for two 90-day periods initially followed by an unlimited number of 60-day periods, although at the beginning of each 60-day period, the physician must again certify that the patient's death is expected within 6 months.

If a patient who has been using heroin is admitted to the psychiatric unit, how long after the last dose was taken is the patient likely to exhibit withdrawal symptoms? 6 to 12 hours 12 to 24 hours 24 to 48 hours 48 to 74 hours

6 to 12 hours Rationale: If a patient who has been using heroin is admitted to the psychiatric unit, the patient is likely to exhibit withdrawal symptoms within 6 to 12 hours after the last dose of heroin because heroin is relatively short acting. Withdrawal symptoms usually peak within 1 to 3 days and subside by the end of a week. Typical withdrawal symptoms include nausea and vomiting, abdominal cramping, myopathy, dysphoric mood, fever, diarrhea, rhinorrhea, and insomnia.

This age range of the elderly is considered young old.

65-74 years

This age range of the elderly is considered Middle old.

75-84 years

This age range of the elderly is considered old old.

>85 years

When instituting suicide precautions, which patient is likely at highest risk? A 15-year-old girl who overdosed on aspirin and then told her best friend. A 50-year-old woman who overdosed on pills and alcohol while her family was present. A 26-year-old man who threatened to jump out of a second-story window. A 38-year-old man who shot himself in the chest while alone at home.

A 38-year-old man who shot himself in the chest while alone at home. Rationale: The patient most likely at risk is the man who shot himself in the chest while alone. A suicide risk assessment should evaluate some of the following criteria: Would the individual sign a contract for safety? Is there a suicide plan? How lethal is the plan? What is the elopement risk? How often are the suicidal thoughts, and has the person attempted suicide before? High-risk findings include: Violent suicide attempt (knives, gunshots). Suicide attempt with low chance of rescue. Ongoing psychosis or disordered thinking. Ongoing severe depression and feeling of helplessness. History of previous suicide attempts. Lack of social support system.

According to Piaget's stages of development, adjusting schemas in response to new information is a process called: Assimilation Accommodation Acclimation Actuation

Accommodation Rationale: According to Piaget's stages of development, adjusting schemas (theories about the manner in which the world functions) in response to new information is a process called accommodation. Applying the schemas to new situations is a process Piaget called assimilation. Piaget believed that there were three tasks that were essential to development and needed to be mastered during childhood: (1) how the world functions, (2) how this functioning is represented in the child's mind, and (3) how this functioning is represented in the minds of others.

In group therapy, a patient who is diagnosed with PTSD begins sobbing and describes a traumatic event that occurred while the patient was in the military and stationed in Afghanistan. According to Yalom's curative factors of group therapy, this is an example of: Universality Catharsis Cohesiveness Imitative behavior

Catharsis Rationale: If, in group therapy, a patient who is diagnosed with PTSD begins sobbing and describes a traumatic event that occurred while the patient was in the military and stationed in Afghanistan, then, according to Irvin Yalom's curative factors of group therapy, this is an example of catharsis. Universality is the recognition that one isn't alone in having problems or discomfort. Cohesiveness is a sense of belonging to a group that is achieving goals, and imitative behavior is a change in behavior brought about by observing others.

A patient has been started on disulfiram before discharge from a treatment facility for alcoholism. A friend comes to visit the patient, and within about 5 minutes of the friend's leaving, the patient becomes hypotensive, complains of severe chest pain, headache, and vertigo, and begins vomiting copiously. The most likely reason is: An adverse reaction to disulfiram Acetaldehyde syndrome Delirium tremens Serotonin syndrome

Acetaldehyde syndrome Rationale: These symptoms are consistent with acetaldehyde syndrome, which occurs if a patient who is taking disulfiram ingests even 7 mL of alcohol. Because the symptoms began shortly after the friend left, the most likely cause is that the friend gave the patient alcohol. Disulfiram alters the metabolism of alcohol so that blood levels rise precipitously within 5 to 10 minutes. The symptoms usually begin to recede within about 30 minutes, but some may persist for several hours.

A patient is able to read printed directions out loud with minimal difficulty. In order to test for comprehension, the psychiatric and mental health nurse should: Give the patient an immediate quiz about the content. Ask the patient to paraphrase the directions. Complete a reading assessment. Give the patient a quiz about the content in 2 days.

Ask the patient to paraphrase the directions. Rationale: If a patient is able to read printed directions out loud with minimal difficulty, this means that the patient is able to decode words. To test for comprehension, after the patient completes the reading, the psychiatric and mental health nurse should ask the patient to paraphrase the directions to determine if the patient was able to understand what was read. Delaying the quiz for 2 days tests retention rather than comprehension. Some people may comprehend well but have poor retention because of impaired memory.

If a patient refuses to take prescribed medications and the psychiatric and mental health nurse threatens to place the patient in restraints and seclusion until the patient cooperates, this may be considered: Battery False imprisonment Assault Malpractice

Assault Rationale: If a patient refuses to take prescribed medications and the psychiatric and mental health nurse threatens to place the patient in restraints and seclusion until the patient cooperates, this may be considered assault, which is an action that results in the patient fearing being touched or handled in an injurious or offensive manner without consent or authority. Battery occurs when harmful or injurious contact occurs. It may or may not result in actual injury. False imprisonment is keeping a patient in unjustified detention. Malpractice is a form of negligence related to professional duties.

A psychiatric/mental health nurse engaged as a consultation liaison is expected to primarily: Meet with consultants in the psychiatric unit. Coordinate activities of different consultants. Serve as a mentor to other nurses in the medical, surgical, and obstetric units. Assess psychiatric disorders and emotional needs of medical, surgical, and obstetric patients.

Assess psychiatric disorders and emotional needs of medical, surgical, and obstetric patients. Rationale: The psychiatric/mental health nurse engaged as a consultation liaison primarily is expected to assess psychiatric disorders and emotional needs of medical, surgical, and obstetric patients. The nurse may assess dementia, confusion, depression, and delirium. Patients are seen upon request of the medical staff primarily responsible for the patient's care, usually when questions about the patient's mental status or ability to make informed decisions arise. In some cases, assessment for suicide or homicide risk or substance abuse may be indicated.

An example of an objective personality test is: Beck Depression Inventory (BDI) Sentence completion test Thematic Apperception Test (TAT) Rorschach test

Beck Depression Inventory (BDI) Rationale: An example of an objective personality test is Beck Depression Inventory (BDI). Objective tests require the person taking the test to choose an answer, either true-false or multiple choice, and do not allow for any free expression. Other objective tests include the Minnesota Multiphasic Personality Inventory (MMPI) and the Tennessee Self-Concept Scale (TSCS). Projective tests, on the other hand, are unstructured and the responses are evaluated by the person administering the test. Projective tests include the Rorschach test, the sentence completion test, and the Thematic Apperception Test (TAT).

With rational emotive behavior therapy (REBT), the model suggests that (A) adversity (activating event) and (C) consequences are strongly influenced by (B): Beliefs about the event Behavior associated with the event Background leading to the event Behavior of others

Beliefs about the event Rationale: With rational emotive behavior therapy (REBT), the model suggests that (A) adversity (activating event) and (C) consequences are strongly influenced by (B) beliefs about the event. According to this model of therapy, people innately have both rational and irrational tendencies, so if the person has irrational beliefs about an activating event, then the consequences result in feelings of defeat, while if the person has rational beliefs, the consequences are more likely to be seen as constructive.

When facilitating change to incorporate evidence-based findings into patient care management, the first step is: Understanding Acting Deciding Believing

Believing Rationale: The first step in facilitating change to incorporate evidence-based findings into patient care management is believing because unless the psychiatric and mental health nurse believes that change is possible, the nurse is defeated before beginning. The next step is to decide on a course of action, considering various options. Next is acting and carrying out the processes of change. This is followed by honestly evaluating the results and, last, acquiring understanding of the process.

Typical behavior associated with intoxication from inhalant use includes: Euphoria, lethargy, impaired judgment, and slurred speech Belligerence, assaultiveness, impaired judgment, and slurred speech Restlessness, nervousness, insomnia, and flushed face Euphoria, anxiety, suspicion, and sensation of slow passage of time

Belligerence, assaultiveness, impaired judgment, and slurred speech Rationale: Typical behavior associated with intoxication from inhalant use includes belligerence, assaultiveness, impaired judgment, and slurred speech. The person may exhibit impaired gait, tremors, and blurred vision. Users may also experience irritation of the mouth and throat. The symptoms may progress to CNS depression and cardiac dysrhythmias, which can result in death. Intoxication occurs within 5 minutes of use and symptoms persist for 60 to 90 minutes. Inhalants are a popular choice for adolescents because of accessibility.

What is the most common eating disorder in the United States?

Binge Eating Disorder (BED)

This theory describes aging as being pre-programmed and irreversible. The life of the cell is pre-determined by the DNA and is on course with its destiny.

Biological programming theory

When using cognitive behavioral therapy for treatment of mood disorders, the focus is on: Rewarding positive behavior Learning self-control Making decisions about care Changing automatic thoughts

Changing automatic thoughts Rationale: When using cognitive behavioral therapy for treatment of mood disorders, the focus is on changing automatic thoughts or thought distortions. Automatic thoughts related to depression include personalizing, all-or-nothing thinking, mind reading, and discounting positives, while automatic thoughts related to mania include the same type of thoughts except that mania involves discounting negatives. Patients' thought processes are challenged and the affected patient is asked to describe evidence for beliefs. Another approach is to help the patient evaluate what would happen if the automatic thoughts were realistic.

A patient is scheduled for electroconvulsive therapy for severe depression unresponsive to antidepressants. The pre-procedure medication that the nurse anticipates giving is a(n): Sedative Analgesic Cholinergic blocking agent Muscle relaxant

Cholinergic blocking agent Rationale: About 30 minutes before electroconvulsive therapy (ECT), the patient usually receives a cholinergic blocking agent, such as atropine sulfate or glycopyrrolate, to decrease secretions, reduce the risk of aspiration, and to increase the heart rate to compensate for the slowing that results from vagal stimulation during ECT. During the procedure, the patient usually receives a short-acting anesthetic (such as methohexital sodium) and a muscle relaxant (succinylcholine chloride).

Which of the following are examples of negative symptoms associated with schizophrenia? Hallucinations and delusions Inappropriate clothing, aggressive behavior, stereotyped behavior Abnormal thought processes and speech patterns Blunt or flat affect, avolition, and reduced speech

Blunt or flat affect, avolition, and reduced speech Rationale: Negative symptoms associated with schizophrenia include blunt or flat affect, lack of energy and passivity (anergia), lack of motivation and inability to initiate tasks (avolition), poverty of speech content and speech production, and sudden interruption in speech and thought patterns so that the patient may stop speaking in the middle of an idea when the patient loses track of what he or she was saying (thought stopping). Negative symptoms impair social functioning and the ability to hold a job because of the patient's difficulty with decision-making and communication.

Which of the following disorders is frequently associated with sexual abuse and incest? Conduct disorder Antisocial personality disorder Bipolar disorder Borderline personality disorder

Borderline personality disorder Rationale: Borderline personality disorder is frequently associated with a history of neglect and abuse, especially sexual abuse and incest. Studies indicate that 20 to 70% of patients with borderline personality disorder report having experienced sexual abuse, but authorities believe the percentage is higher because of patients' reluctance to admit to having been victims of sexual abuse or incest. Borderline personality disorder is characterized by fear of abandonment, unstable interpersonal relationships, poor self-image, impulsivity, suicidal ideation/self-mutilating behavior, affective instability, poor anger control, feeling of emptiness, and dissociative reactions.

Considering para-verbal communication, if a person speaks slowly and in a low-pitched monotone voice, the listener is likely to feel that the speaker is: Bored with the conversation Intelligent and deliberate Confused about the topic of conversation Angry about something

Bored with the conversation Rationale: Considering para-verbal communication, if a person speaks slowly and in a low-pitched monotone voice, the listener is likely to feel that the speaker is bored with the conversation. Para-verbal communication refers to the cadence of speech (slow, fast, deliberate) as well as the tone (low-pitched, high-pitched, monotone, trembling voice) and volume (loud, quiet). Para- verbal communication often communicates the feelings of the speaker, even though that may be unintentional. For example, when people are angry, their speech tends to be louder, more high- pitched, and more rapid.

The medication used to control cocaine craving during withdrawal is: Naloxone Bromocriptine Lorazepam Sodium bicarbonate

Bromocriptine Rationale: Craving for cocaine during withdrawal may be alleviated by bromocriptine (dopamine agonist) 1.5 mg TID. Withdrawal symptoms include cravings, marked depression, suicidal ideation, insomnia, and hyperphagia, so clients must be monitored carefully. Lorazepam is used to control seizures caused by amphetamine use. Antidotes for common drugs and toxins include: Opiates: Naloxone (Narcan®). Toxic alcohols: Ethanol infusion and/or dialysis. Acetaminophen: N-acetylcysteine. Calcium channel blockers, beta-blockers: Calcium chloride, glucagon. Tricyclic antidepressants: Sodium bicarbonate. Ethylene glycol: Fomepizole. Iron: Deferoxamine.

The cardiac abnormality that may be unmasked by treatment with lithium is: Prinzmetal angina Kawasaki disease Brugada syndrome Torsade de pointes

Brugada syndrome Rationale: Brugada syndrome is an autosomal dominant cardiac disorder that can be unmasked by treatment with lithium. Brugada syndrome is characterized by right bundle branch block; ST elevation (V1 to V3) unrelated to cardiac ischemia, electrolyte imbalance, or structural cardiac abnormality; and a normal QT interval. Patients may develop life-threatening cardiac ventricular dysrhythmias and sudden death. Some patients are essentially asymptomatic, but an electrocardiogram shows changes. Other patients may have episodes of syncope and restless sleep and nightmares.

Patients with bipolar disorder are often treated with interpersonal and social rhythm therapy. This therapy helps patients: - Recognize triggers to mood changes -Manage stress -Establish consistent sleep and physical activity schedules -Cope with bipolar disorder

C. Establish consistent sleep and physical activity schedules. Rationale: Interpersonal and social rhythm therapy helps patients with bipolar disorder establish consistent sleep and physical activity schedules. The patients utilize a self-monitoring instrument to monitor their daily activities, including their sleep patterns. Maintaining consistent patterns of activities and sleeping at the same time and for the same duration each night helps to reduce manic and depressive episodes. Patients may also engage in cognitive behavioral therapy, family therapy, and group therapy. If symptoms are severe and the patient does not respond to other treatments, electroconvulsive therapy (ECT) may be considered.

The two primary components of dialectical behavioral therapy are: CBT and group therapy Motivational enhancement therapy and behavior modification CBT and visualization Psychoanalysis and eye movement desensitization and reprocessing therapy

CBT and group therapy Rationale: The two primary components of dialectical behavioral therapy, which is used to treat those with borderline personality disorder, are: CBT (weekly) focuses on adaptive behaviors to help the patient deal with stress/trauma and a prioritized list of problems: suicidal behavior, behavior that interferes with therapy, quality-of-life issues, PTSD response, respect for self, acquisition of behavioral skills, and patient goals. Group therapy (2.5 hours weekly) to learn behavioral skills such as self-distracting and self-soothing.

Which of the following SSRIs should be avoided in patients with congenital long QT syndrome (LQTS)? Fluoxetine (Prozac®) Paroxetine (Paxil®) Sertraline (Zoloft®) Citalopram (Celexa®)

Citalopram (Celexa®) Rationale: The SSRI that should be avoided in patients with congenital long QT syndrome (LQTS) is citalopram because it may cause QT prolongation, and doses of the drug should be limited to no more than 40 mg/day to avoid this adverse effect. Long QT syndrome (congenital or induced), a disruption of the electrical system of the heart, is characterized by irregular cardiac rhythms because depolarization after a contraction is delayed. Patients may develop palpitations and ventricular fibrillation, which can result in death. Long QT syndrome may also result from malnutrition that leads to decreased levels of potassium or magnesium, as may occur with anorexia nervosa.

An appropriate primary intervention for patients at risk of emotional illness resulting from trauma, such as an act of violence, is to: Clarify the patient's problem. Refer for inpatient treatment. Provide behavioral modification therapy. Institute a suicide prevention plan.

Clarify the patient's problem. Rationale: An appropriate primary intervention for patients at risk of emotional illness resulting from trauma, such as an act of violence, is to clarify the patient's problem to ensure that both the patient and the psychiatric and mental health nurse are perceiving the problem in the same manner. Other primary interventions related to trauma include focusing on a reality approach, avoiding lengthy explanations of the problem, helping the patient understand what precipitated the problem, acknowledging the patient's feelings, and showing unconditional acceptance.

When utilizing a cognitive behavioral therapy (CBT) approach with a patient who has anxiety disorder and panic attacks, the psychiatric and mental health nurse asks the patient, "What is the worst thing that can happen to you?" This technique is an example of: Positive reframing Decatastrophizing Thought stopping Assertiveness

Decatastrophizing Rationale: Asking a patient with anxiety disorder and panic attacks, "What is the worst thing that can happen to you?" is an example of decatastrophizing, in which the psychiatric and mental health nurse uses questions to help the patient view the situation more realistically. Thought stopping, forcing oneself to stop thinking about a stressor, can be used to stop negative thoughts. Positive reframing is a technique in which the patient reframes negative thoughts, such as "I'm dying" into more positive thoughts, such as "This is just anxiety and will pass." Assertiveness training may help the patient can confidence.

If a psychiatric and mental health nurse knows the employer of a patient and tells the employer that the patient is too mentally unstable to work, and the patient loses his job as a result, this may constitute: Defamation of character Libel Invasion of privacy Battery

Defamation of Character Rationale: If a psychiatric and mental health nurse knows the employer of a patient and tells the employer that the patient is too mentally unstable to work and the patient loses his job as a result, this may constitute defamation of character since the information was detrimental to the patient's reputation. Defamation of character generally involves accusations that are malicious or false. Sharing information about the patient is a breach of confidentiality. If the nurse had put the information in writing, this would represent libel as opposed to slander, which involves orally giving malicious or false information.

What is the antidote for Iron?

Deferoxamine (desferal) Deferiprone (Ferriprox) Deferasirox (Exjade) for iron overload from blood tranfusion

Patients who are treated with lithium to control the symptoms of bipolar disorder must be advised to avoid: Sun exposure Sodium in the diet Dehydration Tobacco products

Dehydration Rationale: Patients who are treated with lithium to control the symptoms of bipolar disorder must be advised to avoid dehydration because this may cause the blood level of lithium to increase, resulting in toxicity. Patients should drink 8 to 10 glasses of liquid (primarily water) daily and may need increased fluids during hot weather. Patients should not be on a low sodium diet but should maintain a fairly consistent level of sodium intake because lithium levels increase with lower sodium levels and decrease with higher.

If the psychiatric and mental health nurse delegates a task to an unlicensed assistive personnel who states she has no training in the task and doesn't feel comfortable doing it, the most appropriate response is to: Delegate the task to someone else. Report the unlicensed personnel to a supervisor. Assure the unlicensed personnel that the task is easy. Tell the unlicensed personnel that you will check in frequently.

Delegate the task to someone else. Rationale: If the psychiatric and mental health nurse delegates a task to an unlicensed assistive personnel who states she has no training in the task and doesn't feel comfortable doing it, the most appropriate response is to delegate the task to someone else because no unlicensed personnel should be expected to carry out tasks for which they are not trained. However, if the task is one that unlicensed assistive personnel are expected to do, the nurse should later provide or facilitate the needed training.

An acute onset of confusion with fluctuation in attention level and disorganized thoughts or an altered state of consciousness is characteristic of what state of mind?

Delirium

A patient believes that messages are being sent to her in newspapers, magazines, radio, and television and that she must decipher them. This type of delusion is a: Delusion of persecution Delusion of control Delusion of reference Delusion of grandeur

Delusion of reference Rationale: Delusion of reference: Patient believes everything in the environment references her, such as the patient believing that messages are being sent to her in newspapers, magazines, radio, and television and that she must decipher them. Delusion of persecution: Patient believes others intend to harm her. Delusion of control: Patient believes other people or objects control her actions. Delusion of grandeur: Patient believes she is an important person or being, such as God.

A patient who has been diagnosed with bipolar disorder but has consistently refused to take medications or attend therapy, insisting that he has been misdiagnosed and has only "mild stress," is probably experiencing: Dissociation Resistance Denial Suppression

Denial Rationale: A patient who has been diagnosed with bipolar disorder but has consistently refused to take medications or attend therapy, insisting that he has been misdiagnosed and has only "mild stress" is probably experiencing denial, an ego defense mechanism. Denial occurs when a patient refuses to acknowledge a painful truth, such as a diagnosis of bipolar disorder. Denial may also include the failure to recognize the behavior or attitudes that allow problems to continue.

The psychiatric condition that is most commonly associated with hypothyroidism is: Psychosis Depression Bipolar disorder Anxiety

Depression Rationale: The psychiatric symptom that is most commonly associated with hypothyroidism is depression, affecting about 60% of those with the thyroid disorder. Patients may also exhibit emotional lability and alterations in cognition, such as difficulty concentrating and forgetfulness. The degree of depression tends to correlate with an increased thyroid-stimulating hormone level (the normal range is from 0.4 to 4.0 mU/L), but normal values may vary among individuals. Patients with depression should be checked for the possibility of hypothyroidism because thyroid hormone replacement therapy may be more effective than antidepressants in patients with hypothyroidism.

Interpersonal therapy is generally most effective for: Depressive episodes associated with specific situations Major depressive episodes Depressive episodes associated with bipolar disorder Depressive episodes associated with PTSD

Depressive episodes associated with specific situations Rationale: Interpersonal therapy is generally most effective for depressive episodes associated with specific situations (such as grief) and is usually of short duration (six 20-minute sessions). During therapy, the focus is on one issue, such as conflicts, changing roles, or grief. The patient is helped to develop specific goals, and the therapist confronts the patient when the patient's behavior does not facilitate reaching these goals. Patients are encouraged to remain focused on the problem and associated concrete feelings rather than abstract feelings.

The Hamilton Rating Scale for Depression is intended for: Diagnosing depression Self-assessment of depression Determining the severity of diagnosed depression Determining suicidal ideation associated with depression

Determining the severity of diagnosed depression Rationale: The Hamilton Rating Scale for Depression (HAM-D) is completed by the observer and is intended to determine the severity of diagnosed depression. The items on the scale are scored from 0 to 4 or 0 to 2, depending on the nature of the item. The seventeen items included for evaluation of depression include depressed mood; guilt; suicide; initial, middle, and delayed insomnia; work and interest; retardation; agitation: psychic and somatic anxiety; somatic (gastrointestinal); somatic (general); genital; hypochondriasis; insight; and weight loss. Four other items are assessed for general information: diurnal variation, depersonalization, paranoia, and obsessional symptoms.

The Hamilton Rating Scale for Depression is intended for: Diagnosing depression Self-assessment of depression Determining the severity of diagnosed depression Determining suicidal ideation associated with depression

Determining the severity of diagnosed depression Rationale: The Hamilton Rating Scale for Depression (HAM-D) is completed by the observer and is intended to determine the severity of diagnosed depression. The items on the scale are scored from 0 to 4 or 0 to 2, depending on the nature of the item. The seventeen items included for evaluation of depression include depressed mood; guilt; suicide; initial, middle, and delayed insomnia; work and interest; retardation; agitation: psychic and somatic anxiety; somatic (gastrointestinal); somatic (general); genital; hypochondriasis; insight; and weight loss. Four other items are assessed for general information: diurnal variation, depersonalization, paranoia, and obsessional symptoms.

With the four-phase crisis intervention (Aguilera, 1998) method that corresponds to the nursing process, which of the following actions should be completed during Phase IV (evaluation/anticipatory planning)? Identify external support systems (family, friends, social agencies). Discuss how the patient will deal with triggering events in the future. Assess the patient's personal perception of strengths. Identify the precipitating event that caused the crisis.

Discuss how the patient will deal with triggering events in the future. Rationale: 1. Assessment: Gather information about precipitating factors, events, coping methods, support systems, substance abuse, suicide/homicide potential, and the patient's perceptions of strengths. Develop nursing diagnoses. 2. Planning intervention: Select actions for nursing diagnoses. 3. Intervention: Carry out actions, including establishing rapport, setting behavioral limits, clarifying problems, acknowledging feelings, guiding the patient through problem solving, and helping the patient identify new coping mechanisms. 4. Evaluation/Anticipatory planning: Evaluate resolution of crisis and discuss how the patient will deal with triggering events in the future.

A psychiatric and mental health nurse has developed a successful strategy for working with a difficult patient and would like to share this strategy with other team members. The best method is likely to: Ask the supervisor to direct the team to use the strategy. Write out the steps to the strategy and give to each team member. Discuss the strategy during a team meeting. Ask the physician to write the strategy as a physician order.

Discuss the strategy during a team meeting. Rationale: If a psychiatric and mental health nurse has developed a successful strategy for working with a difficult patient and would like to share this strategy with other team members, the best method is likely to discuss the strategy during a team meeting rather than trying to impose the strategy on others without discussion. During discussion, the nurse may discover that others have also devised successful strategies and have input about strategies that are less successful.

A patient with advanced Parkinson's disease states that a strange, unrecognizable animal keeps appearing in her yard for brief periods and then disappears. The patient insists that she is alert and awake during these occurrences, but other family members deny observing the animal. The initial intervention that is indicated is to: Discuss visual hallucinations, reassure the patient, and teach coping methods. Advise the patient to ask a family member to confirm the sightings. Provide a detailed explanation of pathological changes that cause hallucinations. Tell the patient not to worry because the "animal" does not appear to pose a threat.

Discuss visual hallucinations, reassure the patient, and teach coping methods. Rationale: If a patient with advanced Parkinson's disease states that a strange, unrecognizable animal keeps appearing in her yard for brief periods and then disappears and insists that she is alert and awake during these occurrences, but other family members deny observing the animal, the initial intervention that is indicated is to discuss visual hallucinations (which are fairly common in advanced Parkinson's disease); reassure the patient that the hallucinations will pass; and teach coping methods, such as looking away, reminding the self that it is a hallucination, and focusing attention elsewhere.

A patient who lost his job because of his inability to complete his work tasks yells at the psychiatric and mental health nurse that she is "mean and stupid" and ruining his life. Which ego defense mechanism is the patient using? Identification Displacement Sublimation Projection

Displacement Rationale: Displacement: Expressing strong feelings generated by one person to another who is less threatening. In this case, yelling at the nurse instead of the boss who fired him. Identification: Modeling behavior or attitudes on those of another, such as entering the same profession as a mentor. Sublimation: Substituting behavior that is acceptable for one that is not, such as chewing gum instead of smoking. Projection: Unconsciously blaming unacceptable feelings/actions on someone else, such as by attacking gay people to deny homosexual attraction.

A patient whose husband died in a car accident eight months earlier is in a deep state of despair and is unable to function in normal activities. She has exaggerated expressions of anger, sadness, and guilt and often blames herself. This type of grief is: Prolonged Inhibited Distorted Anticipatory

Distorted Rationale: Distorted grief, which results in severe despair, inability to function, exaggerated expressions of grief (anger, sadness, guilt), and self-blame, is a maladaptive grief response. Prolonged grief may persist for years with the person vacillating between anger and denial. Inhibited/Delayed grief occurs when the person is not able to get past the denial stage of grief and cannot come to emotional terms with the death. Anticipatory grief is grieving that occurs before an anticipated loss, such as when a partner is nearing death.

An appropriate nursing diagnosis for a patient who experiences delusional thinking, decreased volition, impaired problem solving, impaired abstract thinking, and suspicion of others is: Disturbed sensory perception Social isolation Disturbed thought processes Self-care deficit

Disturbed thought processes Rationale: Disturbed thought processes: Patient cannot trust and may experience panic with behavior that may include delusional thinking, decreased volition, impaired problem solving, impaired abstract thinking, and suspicion of others. Disturbed sensory perception: Patient perceives incoming stimuli abnormally and exhibits distorted response. Social isolation: Patient has impaired or absent ability to interact with others. Self-care deficit: Patient has impaired or absent ability to carry out self-care activities, such as bathing.

A male patient who acts very polite and considerate in the presence of other patients and staff makes threatening and vulgar sexual comments to the psychiatric and mental health nurse when no one else can hear. The most appropriate method of reporting this behavior is to: File an incident report. Document exactly what the patient said and the circumstances in the nursing notes. Document that the patient was "behaving inappropriately" in the nursing notes. Report the situation to the administrator.

Document exactly what the patient said and the circumstances in the nursing notes.

Which of the following is a characteristic of binge-eating disorder (BED)? Eating more slowly than usual Eating alone out of embarrassment at the amount of food eaten Eating large amounts of food in response to hunger Eating large amounts but never feeling full

Eating alone out of embarrassment at the amount of food eaten Rationale: Characteristics of binge-eating disorder (BED), which is regular episodes of overeating, include: Eating alone out of embarrassment at the amount of food eaten. Eating large amounts of food despite no feeling of hunger. Eating large amounts of food despite feeling uncomfortably full. Eating very rapidly. Having feelings of self-disgust and guilt over eating habits. BED is newly included in the DSM-5 as a distinct disorder and is considered the most common eating disorder in the United States.

A patient who has gone through many relationships and sees positive qualities in the partner initially but as time passes becomes more and more focused on the person's negative qualities or perceived faults and ends the relationship is exhibiting: Reaction formation Introjection Splitting Rationalization

Splitting Rationale: A patient who has gone through many relationships and sees positive qualities in the partner initially but as time passes becomes more and more focused on the person's negative qualities or perceived faults and ends the relationship is exhibiting splitting. Splitting occurs when the patient is unable to integrate the positive and negative aspects of the self or others and tends to view the self or another person as all good or all bad with nothing in-between.

Which of the following herbal preparations should be avoided with other psychoactive drugs? Chamomile Ginseng Fennel St. John's wort

St John's Wort Rationale: St. John's wort, which is used to treat mild to moderate depression, should be avoided with other psychoactive drugs. St. John's wort may increase symptoms of ADHD if patients are taking methylphenidate. St. John's wort may also increase episodes of mania in patients with bipolar disorder and may increase risk of developing mania in those with major depression. St. John's wort may trigger psychosis in some patients with schizophrenia. St. John's wort is associated with many drug interactions, including alprazolam (Xanax®), birth control pills, phenobarbital, phenytoin, amitriptyline (Elavil®), and SSRI.

If a patient's nursing diagnosis is "risk for other-directed violence," an immediate expected outcome of intervention is that the patient will: Exercise control over his emotions. Refrain from hurting others. Express feelings in a non-threatening manner. Identify methods to relieve aggressive feelings.

Refrain from hurting others. Rationale: If a patient's nursing diagnosis is "risk for other-directed violence," an immediate expected outcome of intervention is that the patient will refrain from hurting others. Other outcomes that should be immediate include refraining from destroying property and demonstrating decreased acting out behavior, restlessness, fear, anxiety, and hostility. Patients may need more time and therapy to be able to exercise control over emotions, express feelings in a non-threatening manner, and identify methods to relieve aggressive feelings.

A patient who is fearful about interacting with others attempts to change her perception by imagining that the others are afraid to interact with her. The strategy that the patient is utilizing is: Assertiveness Reframing Repressive coping Intellectualism

Reframing Rationale: A patient who changes her perceptions and the way she thinks to allow her to cope with and overcome stressors is utilizing reframing. Reframing can be helpful if the situation is one that the person can think about in advance in order to develop a more creative frame of mind; however, if the event/situation is so stressful that the person cannot bear to even think about it, then this technique is not likely to be effective.

The primary advantage of case management for community care of a patient with severe mental health issues is that case management: Is more cost-effective than hospitalization Eases the burdens of other care providers Relieves the patient of the responsibility to coordinate and manage care Allows insurance companies to better determine allowable coverage for services

Relieves the patient of the responsibility to coordinate and manage care Rationale: The primary advantage of case management for community care of a patient with severe mental health issues is that case management relieves the patient of the responsibility of coordinating and managing care, especially those patients with limited support systems. Patients may easily feel overwhelmed if they have to access services from a number of different resources and may fail to follow through with the care plan, resulting in recurrence or exacerbation of symptoms.

A characteristic of active listening includes: Talking during periods of speaker silence Remaining nonjudgmental Offering opinions Focusing on details

Remaining nonjudgmental Rationale: A characteristic of active listening includes remaining nonjudgmental. Active listening involves focusing on the individual and what the individual is saying and avoiding unnecessary interruptions, such as asking questions and giving opinions and advice. The listener should avoid filling silences with words but should remain patient and give verbal and nonverbal signs that the listener is paying attention (nodding the head, "yes," "I see," mirroring body language), restating, and summarizing to ensure that the listener has understood. The listener should focus on the overall message more than on the details.

If a patient with severe postpartum depression admits she hates her infant but states, "I would never hurt it," the first priority should be to: Encourage the patient to ask for help with childcare. Advise the patient's husband to monitor childcare. Remove the infant from the patient's care. Advise the patient to find a family member to care for the child.

Remove the infant from the patient's care. Rationale: If a patient with severe postpartum depression admits she hates her infant but states "I would never hurt it," the first priority should be to remove the infant from the patient's care because the patient has admitted hating the child and has depersonalized the child by referring to the child as "it." Additionally, a patient with severe postpartum depression is at risk for postpartum psychosis, which may further increase risk to the infant.

If a 30-year-old patient with paranoia and schizophrenia states he does not want his parents (who are paying for his care) to visit because he believes they are "possessed by devils," the psychiatric and mental health nurse should: Ask the physician to intervene. Allow the parents to visit. Respect the patient's request. Suggest the parents get a court order to allow visits.

Respect the patient's request. Rationale: If a 30-year-old patient with paranoia and schizophrenia states he does not want his parents (who are paying for his care) to visit because he believes they are "possessed by devils," the psychiatric and mental health nurse should respect the patient's request. Patients' rights are not determined by who is paying for care but remain with the person. Unless the patient has been declared incompetent in a court proceeding and his parents granted conservatorship, the patient can deny them visitation.

The mother of an adolescent with autism spectrum disorder with severe impairment states she is often so tired at the end of the evening that she breaks down and cries. The care support that is probably the most essential at this time is: Respite care Support group Volunteer visitor Spiritual support

Respite Care Rationale: Caregiving can be exhausting, so if the mother of an adolescent with autism spectrum disorder with severe impairment is so tired that she begins crying, then she is overwhelmed and is most in need of respite care. The caregiver needs a break of even a few days in order to rest and have time for herself. If this is not possible, then part-time respite care in the home to allow the caregiver to relinquish caregiving for a few hours may help to reduce stress.

An 8-year-old boy has been sleeping poorly, complaining of stomachaches, crying frequently, and refusing to go to school. A complete physical examination ruled out a physical ailment. As part of an assessment for anxiety, the simplest assessment tool to use with a child is the: Hamilton Anxiety Scale (HAS) [Or the more common acronym "HAMA"] Beck Anxiety Inventory (BAI) Beck Depression Inventory (BDI) Revised Children's Manifest Anxiety Scale (RCMAS)

Revised Children's Manifest Anxiety Scale (RCMAS) Rationale: RCMAS assesses anxiety in children and adolescents (6-19) with 37 yes-no questions and can be read to young children. HAS [or the more common acronym "HAMA"] provides an evaluation of overall anxiety and its degree of severity for children and adults. This scale is frequently utilized in psychotropic drug evaluations. BAI is a tool for adolescents and adults that ranks 21 common symptoms related to anxiety according to the degree they have bothered the client in the previous month. BDI is a widely utilized, self-reported, multiple-choice questionnaire consisting of 21 items, which measures the degree of depression for those 17 to 80 years.

The treatment of choice for generalized anxiety disorder (GAD) in older adults is: Benzodiazepine Tricyclic antidepressants SSRI Alpha-adrenergic agonist

SSRI Rationale: The treatment of choice for generalized anxiety disorder (GAD) in older adults is an SSRI. Doses are usually started at a lower level than for younger adults as high doses may increase anxiety. In older adults, late onset GAD and panic attacks (less common) are most often associated with depression and, in some cases, physical illness, such as heart disease. As well as GAD, agoraphobia and other phobias are common conditions associated with anxiety in older adults.

When working with a patient with conduct disorder, limit setting includes (1) informing patient of limits, (2) explaining the consequences of noncompliance, and (3): Providing feedback Stating reasons Establishing time limits Stating expected behaviors

Stating expected behaviors Rationale: When working with a patient with conduct disorder, limit setting includes (1) informing patient of limits, (2) explaining the consequences of noncompliance, and (3) stating expected behaviors. Application of limit setting must be consistent and carried out by all staff members at all times. Consequences must be individualized and must have meaning for the patient so that the patient is motivated to avoid them. Negotiating a written agreement that can be referred to can prevent conflicts if the patient tries to change the limits.

Which of the following organizations/agencies provides the Evidence-Based Practices (EBP) Web Guide for treatment of mental and substance abuse disorders? Substance Abuse and Mental Health Services Administration (SAMHSA) United States Food and Drug Administration (FDA) National Alliance on Mental Illness (NAMI) American Psychological Association (APA)

Substance Abuse and Mental Health Services Administration (SAMHSA)

If a family member of a patient with mental health and substance abuse issues is seeking help and information and is unsure of whom to call or where to go, the best resource is likely the: American Psychological Association (APA) American Association for Suicidology (AAS) Substance Abuse and Mental Health Services Administration (SAMHSA) National Association for Rural Mental Health (NARMH)

Substance Abuse and Mental Health Services Administration (SAMHSA) Rationale: If a family member of a patient with mental health and substance abuse issues is seeking help and information and unsure whom to call or where to go, a good resource is SAMHSA, which provides a national helpline (1-800-662-HELP) and provides referrals to local programs as well as information about different treatments, disorders, and preventive measures such as "Help prevent suicide."

A patient with opioid use disorder is to be maintained as an outpatient on Suboxone® (buprenorphine plus naloxone). The psychiatric and mental health nurse expects that the patient will begin with: Suboxone®, with first administration 24 hours after last opioid Subutex® for one day and then switch to Suboxone® Suboxone®, with first administration immediately after last opioid Subutex® (buprenorphine only) for two days and then switch to Suboxone®

Subutex® (buprenorphine only) for two days and then switch to Suboxone® Rationale: If a patient with opioid use disorder is to be maintained as an outpatient on Suboxone® (buprenorphine/naloxone), the patient will usually begin with Subutex® (buprenorphine only) for two days and then switch to Suboxone®. Subutex® is initiated when the patient begins experiencing withdrawal symptoms (≥4 hours after last narcotic dose) as the drug helps to reduce cravings and prevent withdrawal symptoms. Suboxone® contains the opioid antagonist naloxone, which may cause severe withdrawals if the patient has not been free of narcotics. If a patient takes narcotics while on Suboxone®, the patient will experience immediate withdrawal, and this provides some insurance against drug abuse.

A 16-year-old adolescent is being treated with fluoxetine (a selective serotonin reuptake inhibitor [SSRI]) and cognitive behavioral therapy (CBT) for severe anxiety and depression 6 months after the death of her mother. The patient must be monitored and regularly assessed for: Substance abuse Polypharmacy Suicidal ideation Noncompliance

Suicidal Ideation Rationale: Adolescents respond well to a combination of a selective serotonin reuptake inhibitor (SSRI) and cognitive behavioral therapy (CBT) for the treatment of depression, but SSRI use in adolescents has been associated with increased suicidal ideation, so the patient must be carefully monitored and assessed. The patient and their family should be educated about this possible effect and warning signs of suicidal ideation. In some cases, adolescents may be asked to sign a no-suicide contract that clearly outlines the steps to take in the event that they feel suicidal.

The greatest concern(s) for a patient with postpartum psychosis is (are): Suicide and/or infanticide Self-mutilation Family abandonment Family abuse (spouse, children)

Suicide and/or infanticide Rationale: The greatest concerns for a patient with postpartum psychosis are suicide and/or infanticide. The onset of psychosis is often abrupt and may occur within 24 hours to 2 weeks of delivery. Symptoms may begin with irritability and difficulty sleeping but may rapidly progress to disorientation and erratic behavior. Delusions and auditory hallucinations may occur. A priority intervention is to remove the infant from the patient's care until the patient's condition stabilizes.

At the end of a discussion with a patient about modifying the patient's plan of care, the psychiatric and mental health nurse states: "I understand you to say that you want to try some alternative treatments, such as imagery and relaxation, to help cope with your anxiety." This is an example of: Validating Summarizing Restating Assessing

Summarizing Rationale: If a psychiatric nurse ends a discussion with the patient about modifying the patient's plan of care by saying, ""I understand you to say that you want to try some alternative treatments, such as imagery and relaxation, to help cope with your anxiety," this is an example of summarizing. With summarizing, it's important to accurately reflect the patient's statements without judgment. Stating the summary verbally helps to verify that the nurse's understanding is correct and helps the patient feels the patient's ideas are validated.

A patient has been prescribed chlorpromazine hydrochloride for psychosis and is to continue on the oral medication after discharge. As part of educating the patient about the medication, the psychiatric and mental health nurse should advise the patient to avoid: Exercise Milk products Tobacco products Sun exposure

Sun Exposure Rationale: If a patient has been prescribed chlorpromazine hydrochloride, an antipsychotic, the nurse should advise the patient to avoid sun exposure, as photosensitivity reactions of the skin may occur. When outside or exposed to sun, patients should wear sun block and protective clothing. Chlorpromazine is more likely to cause photosensitivity than other medications in the same class. Patients should also avoid alcohol while taking the drug and should avoid activities that require good coordination, especially in the initial weeks of therapy.

If a 27-year-old patient with narcissistic personality disorder is pregnant and has made plans to have an abortion but the psychiatric and mental health nurse is opposed to abortion for religious reasons, the nurse should: Discuss alternatives with the patient. Provide literature about adoption. Advise the patient her decision is morally wrong. Support the patient's decision.

Support the patient's decision. Rationale: If a 27-year-old patient with narcissistic personality disorder is pregnant and has made plans to have an abortion but the psychiatric and mental health nurse is opposed to abortion for religious reasons, the nurse should support the patient's decision. The patient has the legal right to make this decision, and the nurse must use care not to impose personal religious beliefs onto the patient or try to pressure the patient into making a different decision.

If a patient with post-traumatic stress disorder (PTSD) is invited to a wedding but, when inside the church, begins to experience a flashback to an explosion inside of a building, a grounding method that may help is for the patient to: Get up and leave immediately Take an inventory of things inside the church Repeat the phrase "This is just a church" over and over Close the eyes and visualize another place

Take an inventory of things inside the church Rationale: If a patient with PTSD is invited to a wedding but, when inside the church, begins to experience a flashback to an explosion inside of a building, a grounding method that may help is for the patient to take an inventory of things inside of the church, such as the number of flowers or colors in the room or the number of males and females. This type of concentration may help to focus the attention away from the flashback.

A patient presents with a changing topic of discussion to an entirely different topic, how the patient never returns to the original topic and does not answer the original question. What thought process is this patient presenting?

Tangential

If the psychiatric and mental health nurse asks a patient a question and the patient wanders completely off topic in the response and never answers the questions, this is an example of: Loose association Word salad Flight of ideas Tangential thinking

Tangential thinking Rationale: Tangential thinking: The patient wanders completely off topic in responding to a question and never actually answers the questions. Loose association: The patient jumps haphazardly from one idea to another with no obvious association between the various thoughts expressed. Word salad: The patient uses a stream of completely unconnected words that express no meaning. Flight of ideas: The patient speaks rapidly, using many words, but ideas are fragmented and unrelated to each other.

A patient who has received long-term treatment with haloperidol as an antipsychotic agent has developed repetitive behaviors, including tongue thrusting, lip smacking, and hair pulling. The most likely cause is: Pseudoparkinsonism Serotonin syndrome Neuroleptic malignant syndrome Tardive dyskinesia

Tardive dyskinesia Rationale: If a patient has received long-term treatment with haloperidol as an antipsychotic agent and has developed repetitive behaviors, including tongue thrusting, lip smacking, and hair pulling, the most likely cause is tardive dyskinesia (TD), which is related to use of conventional antipsychotics. TD may result in permanent non-voluntary movements. Symptoms are often irreversible but stopping or changing the medication may reduce symptoms. Patients must be monitored carefully for initial indications of TD so that medications can be changed before symptoms become severe.

When the psychiatric/mental health nurse delegates a task to another healthcare provider, the most important consideration when choosing the right person is: Availability Reliability Education/skills Years of experience

Education/skills Rationale: The most important consideration is the education and skills of the person to whom the task is delegated. Five rights of delegation include: Right task: Determine an appropriate task to delegate for a specific patient. Right circumstance: Consider the setting, resources, time factors, safety factors, and all other relevant information to determine the appropriateness of delegation. Right person: Choose the right person (by virtue of education/skills) to perform a task for the right patient. Right direction: Provide a clear description of the task, the purpose, any limits, and expected outcomes. Right supervision: Supervise, intervene as needed, and evaluate performance of the task.

This is the conscious mind, rational mind, reality based, employs defense mechanisms --Thinking Process --Reality Tests --Employs defense mechanisms as a means of protecting the integrity of the mind

Ego-Sigmund Freud

A patient becomes overwhelmed when asked to make even a simple decision, such as choosing between an egg and toast or cereal for breakfast. This probably results from: Depersonalization Anhedonia Regression Emotional ambivalence

Emotional ambivalence Rationale: Emotional ambivalence: Patient has opposite emotions about the same thing and cannot, therefore, make even a simple decision, such as choosing between an egg and toast or cereal for breakfast. Depersonalization: Patient experiences feelings of unreality, such as seeing the self from a distance or perceiving distortion in the body. Anhedonia: Patient is unable to experience pleasure. Regression: Patient retreats to an earlier stage of development as a coping mechanism.

Bandura's Social Cognitive Theory

Emphasizes behavior, environment, and cognition as the key factors in development

If a patient with alcohol abuse disorder has started taking disulfiram but begins to drink while taking the medication, the patient should expect to: Become rapidly intoxicated Suffer no adverse effects Experience no effect from the alcohol Experience severe adverse effects

Experience severe adverse effects Rationale: If a patient with alcohol abuse disorder has started taking disulfiram but begins to drink while taking the medication, the patient should expect to experience severe adverse effects, such as difficulty breathing, nausea, vomiting, insomnia, diarrhea, and anxiety. The medication should not be taken until five days after the last intake of alcohol. The medication may cause abnormal thoughts, depression, amnesia, and suicidal ideation, so the patient must be monitored carefully

A patient who is severely claustrophobic and is undergoing initial treatment is asked to stay for a specified period of time inside a small, enclosed room. This is an example of the technique of: Flooding Systematic desensitization Implosive therapy Aversion therapy

Flooding Rationale: If a patient who is severely claustrophobic and is undergoing initial treatment is asked to stay for a specified period of time inside a small, enclosed room, this is an example of flooding. Flooding is an exposure therapy used primarily to help control phobias. Flooding is used as an initial aspect of therapy rather than as a more gradual approach to desensitization. The theory is that the anxiety that the patient experiences will eventually lessen, which helps to reduce fear, although not all patients can tolerate flooding.

What is the antidote for ethylene glycol?

Fomepizole

What psychological and social-cultural factors may trigger aggressive behavior?

Frustration (frustration-aggression principle), previous reinforcement for aggressive behavior, observing an aggressive role model, and poor self-control can all contribute to aggression. Media portrayals of violence provide social scripts that children learn to follow. Viewing sexual violence contributes to greater aggression toward women. Playing violent video games increases aggressive thoughts, emotions, and behaviors.

In a group process, the three major types of roles that group members assume within the group are (1) completing group tasks, (2) supporting the group process, and (3): Fulfilling personal needs Controlling the group Challenging the group Providing moral guidance

Fulfilling personal needs Rationale: In a group process, the three major types of roles that group members assume within the group are: Completing group tasks: Roles may include coordinating, evaluating, energizing, orienting, and elaborating. Supporting the group process: Roles may include compromising, encouraging, following, Supporting the group process: Roles may include compromising, encouraging, following, harmonizing, and gate keeping. Fulfilling personal needs: Roles may include being aggressive, dominating, blocking, help- seeking, monopolizing, seeking recognition, and seducing. While completing group tasks and supporting group processes function to make the group effective, roles involved in fulfilling personal needs may interfere with the overall functioning of the group.

During the initial assessment, a 75-year-old female states she has had one fall in the last 4 months but had no residual injury. What, if any, further testing is immediately indicated? No further testing Gait, balance, and get-up-and-go X-ray of hips and spine Bone mass density testing

Gait, balance, and get-up-and-go Rationale: According to the American Geriatrics Society Guideline for the Prevention of Falls in Older Persons, if a patient has one fall, the patient should be assessed for gait and balance, including the get-up-and-go test in which the patient stands up from a chair without using arms to assist, walks across the room, and returns. If the patient is steady, no further assessment is needed. If the patient demonstrates unsteadiness, further assessment to determine the cause is necessary.

When evaluating outcome data for evidence-based practice, the type of data that includes measures of mortality, longevity, and cost-effectiveness is: Clinical Psychosocial Integrative Physiological

Integrative Rationale: A number of different types of outcome data must be considered when evaluating outcomes data Integrative (includes measures of mortality, longevity, and cost-effectiveness). Clinical (includes symptoms, diagnoses, staging of disease, and indicators of individual health). Physiological (includes measures of physical abnormalities, loss of function, and activities of daily living). Psychosocial (includes feelings, perceptions, beliefs, functional impairment, and role performance). Perception (includes customer perceptions, evaluations, and satisfaction). Organization-wide clinical (includes re-admissions, adverse reactions, and deaths).

A patient whose partner has left him for someone else and who spends an hour discussing all of the positive aspects of being single is probably utilizing the ego defense mechanism of: Displacement Intellectualism Denial Rationalization

Intellectualism Rationale: Intellectualism: Using rational intellectual processes to deal with stress and loss, such as by discussing positive aspects of being single. Displacement: Transferring feelings from one person or thing to another, such as being angry with a boss and taking the anger out on a spouse. Denial: Completely refusing to acknowledge a situation that is stressful, such as ignoring a child's drug use. Rationalization: Attempting to find excuses for unacceptable behavior or feelings, such as drinking to relieve the stress of work.

A patient whose partner has left him for someone else and who spends an hour discussing all of the positive aspects of being single is probably utilizing the ego defense mechanism of: Displacement Intellectualism Denial Rationalization

Intellectualism Rationale: Intellectualism: Using rational intellectual processes to deal with stress and loss, such as by discussing positive aspects of being single. Displacement: Transferring feelings from one person or thing to another, such as being angry with a boss and taking the anger out on a spouse. Denial: Completely refusing to acknowledge a situation that is stressful, such as ignoring a child's drug use. Attempting to find excusRationalization: es for unacceptable behavior or feelings, such as drinking to relieve the stress of work.

With cognitive behavioral therapy (CBT), a technique that is used for the treatment of anxiety and panic attacks is: Interoceptive exposure Exposure and response prevention Play the script until the end Unraveling cognitive distortions

Interoceptive exposure Rationale: With CBT, a technique that is used for the treatment of anxiety and panic attacks is interoceptive exposure. This technique requires putting the patient into a situation in which the patient experiences the fear and body sensations, such as shortness of breath or chest tightness, so the patient can learn that although the symptoms are frightening and uncomfortable at the time, they are not dangerous or life-threatening.

If the psychiatric and mental health nurse overhears other staff beginning to discuss difficulties caring for an unnamed patient in the staff dining room where other staff are present, the nurse should: Intervene to tell staff that their comments can be overheard. Reprimand the staff for violating privacy. Take no action as the patient was unnamed. Report the violation of privacy to a supervisor.

Intervene to tell staff that their comments can be overheard. Rationale: If the psychiatric and mental health nurse overhears other staff beginning to discuss difficulties caring for an unnamed patient in the staff dining room where other staff is present, the nurse should intervene to tell staff that their comments can be overheard. Staff members often discuss patient care issues over lunch or breaks without considering that others may overhear. It is a violation of privacy whether or not the patient is named because some identifying information (age, gender, diagnosis) may be divulged unintentionally.

Which of the following ethnic groups is most likely to believe that mental illness is the result of a loss of self-control or punishment for bad behavior? Mexican Americans Japanese Americans Puerto Ricans Chinese

Japanese Americans Rationale: The ethnic group that is most likely to believe that mental illness is the result of a loss of self- control or punishment for bad behavior is Japanese American. Puerto Ricans often believe that mental illness results from heredity or from prolonged suffering. Chinese are more likely to believe that mental illness results from evil spirits or a lack of harmony in emotions. Mexican Americans attribute mental illness to a variety of causes, including God, spirituality, and interpersonal relationships.

Which of the following best describes Kolb's model of experiential learning? Knowledge develops from experience interacting with cognition and perception. Knowledge and experience are equally important. Experience precedes knowledge in learning. Learning cannot be acquired without experience and perception.

Knowledge develops from experience interacting with cognition and perception. Rationale: Kolb's model of experiential learning is based on acquiring knowledge through grasping experience and transforming that experience into knowledge through cognitive processes and perception. Experience may be transformed into knowledge through abstract conceptualizing (analyzing, thinking), observing others, or actively experimenting. This model stresses that the individual makes choices between the concrete and the abstract, and this is reflected in learning styles: Diverging: Concrete experience and reflective observation Assimilating: Abstract conceptualization and reflective observation Converging: Abstract conceptualization and active experimentation Accommodating: Concrete experience and reflective observation

Which of the following could be an example of elder neglect? Insulting, name-calling Lack of dentures Physically restraining the patient Misusing patient's financial resources

Lack of Dentures Rationale: Lack of dentures, hearing aids, or glasses may be examples of elder neglect, which may be intentional or unintentional. However, one cannot jump to conclusions. For example, the patient may have refused to wear dentures or may be unable to afford them. Other signs of neglect may include inadequate access to food or fluids, inadequate heating or air conditioning, unclean personal belongings/clothes, and lack of necessary medications. Insulting, name calling, physically restraining the patient, and misusing the patient's financial resources are indications of abuse.

The ethnic/cultural group in the United States that is most likely to attribute a psychiatric disease to a medical illness rather than a mental illness is: African Americans Caucasians Asian Americans Latinos

Latinos Rationale: The ethnic/cultural group in the United States that is most likely to attribute a psychiatric disease to a medical illness rather than a mental illness is Latinos. Only about a third of Latinos seek treatment for mental illness, which is a lower percentage than other groups. In Latino communities, a stigma against mental illness persists, making it difficult for people to seek treatment.

Mike Brown has completed gender reassignment surgery (male-to-female) and is now legally Mikaela Brown. Mikaela states that she is still attracted to females and not males. Her sexual orientation should be most appropriately classified as: Lesbian Heterosexual Homosexual Bisexual

Lesbian Rationale: Once a person completes gender reassignment surgery and legally changes genders, that person is then considered the reassigned gender; thus, Mikaela is considered female, so her attraction to other females would result in her sexual orientation as lesbian. If she were attracted to males, she would be heterosexual. While she could also be classified as homosexual, this term is more commonly used for gay males, and she is no longer considered a male. She does not report a bisexual attraction to both genders.

A patient taking clozapine for treatment should be regularly monitored for: Liver function Leukopenia Renal function Hypoglycemia

Leukopenia Rationale: A patient taking clozapine for treatment should be regularly monitored for leukopenia and agranulocytosis with regular WBC counts. Clozapine is recommended for patients with schizophrenia not responding to other treatments and who are at risk for suicide. Clozapine has significant adverse effects, including seizures, cardiomyopathy, myocarditis, pulmonary embolism, and cardiac arrest. The drug should be used with caution with benzodiazepines and other psychotropic drugs because the combination may result in respiratory arrest. Clozapine has multiple drug-drug interactions so all medications should be carefully reviewed before beginning treatment.

A 30-year-old patient with a 6-year-old child is treated in an outpatient clinic. The patient often has little money for food and purchases inexpensive non-nutritious fast food. The most appropriate recommendation is: Home delivery of meals (Meals-on-Wheels) Nutritional counseling WIC Local food banks

Local food banks Rationale: While nutritional counseling may be of value, the immediate needs relate to lack of money, so the most appropriate recommendation is to local food banks, which should be able to provide groceries to those with low income. Home delivery of meals is usually restricted to the elderly or disabled and not available to the general public. WIC programs serve mothers up to a year after delivery if breastfeeding and children until their fifth birthday, so this family is no longer eligible.

For patients with dissociative amnesia, the type of amnesia that involves the inability to recall a traumatic event for a few hours or few days after the event is classified as: Localized Selective Generalized Systematized

Localized Rationale: Dissociative amnesia: Localized: Inability to recall a traumatic event for a few hours or few days after the event. Selective: Inability to recall some aspects of a traumatic event for a period after the trauma.Generalized: Inability to recall any of previous history, including identity. Systematized: Inability to recall a specific category of information or a specific person or event. Continuous: Inability to recall events after a specific time until the present.

What is a personality disorder?

Longstanding, pervasive, inflexible, extreme, and persistent patterns of behavior and inner experience

During the initial patient interview, the patient jumps from one topic to another with little coherence. The term for this type of communication is: Word salad Flight of ideas Evasion Loose association

Loose Association Rationale: Loose association is jumping from one topic to another with little coherence although the content is understandable. Word salad is a flow of incoherent disconnected words that do not convey meaning. Flight of ideas is speech that is excessive and rapid, with unrelated ideas: "The car is new. The dog barked all night, and the moon was out. I need some water." Evasion is avoidance of making clear statements: "I ... um ... think ... that could be ... perhaps ... right." Neologism is the creation of new words.

A patient tends to walk and sit in a slumped over position, keeping the head and eyes down, and rarely making eye contact or initiating conversation. The patient responds with a low tone of voice. These behaviors most likely represent: Dislike Low self-esteem Fear Anxiety

Low self-esteem Rationale: While patients may vary in how they express feelings, walking or sitting slumped over, keeping the eyes and head down, avoiding eye contact, failing to initiate conversations, and keeping a low tone of voice are consistent with low self-esteem. The psychiatric and mental health nurse should help the patient to establish realistic and achievable goals and help the patient identify positive personal aspects and strengths. The patient may need one-on-one support in difficult situations, such as group therapy, to deal with fear of failure.

Which of the following diets may decrease excretion of lithium and cause toxic levels to develop? Low fat Low protein Low carbohydrate Low sodium

Low sodium Rationale: Lithium is administered as a lithium salt in the form of lithium carbonate or lithium citrate. The sodium in lithium attaches more readily to sodium receptors on cells than the sodium in sodium chloride (table salt), so low sodium intake prevents adequate uptake. Additionally, excretion of lithium is associated with excretion of sodium, so hyponatremia results in decreased excretion of lithium, causing toxic levels to develop. Patients on lithium must have adequate sodium and fluid intake.

If a patient is a very heavy smoker and has tried to quit cold turkey many times but has failed each time, the initial intervention that is likely to be most effective is: Nicotine lozenges Nicotine patch Nicotine gum Varenicline (Chantix).

Nicotine Patch Rationale: If a patient is a very heavy smoker and has tried to quit cold turkey many times but has failed each time, the initial intervention that is likely to be most effective is the nicotine patch because it provides a steady rate of nicotine absorption to help control cravings. A patch is applied one time daily. Adverse effects may include an allergic response, skin irritation, insomnia, and tachycardia. Some patients supplement the patch with nicotine lozenges to control intense cravings.

To ensure that a patient has taken an oral medication, the psychiatric and mental health nurse should: Observe the patient taking the medication and check the open mouth. Observe the patient taking the medication followed by 60 mL of fluid. Mix the medication with food and administer. Observe the patient taking the medication followed by supervision for 5 minutes.

Observe the patient taking the medication followed by 60 mL of fluid. Rationale: To ensure that a patient has taken an oral medication, the psychiatric and mental health nurse should observe the patient taking the medication followed by at least 60 mL of fluid. If the hoarding of medications is a concern, then the patient's mouth may be checked. Medications should not be mixed with food or drink for administration unless there is a valid reason that is documented, such as the inability to swallow pills or confusion, because doing so may constitute coercion.

A 27-year-old male with a history of drug and alcohol abuse, stealing, and bullying behavior has been working with the psychiatric/mental health nurse on educational issues related to substance abuse, self- esteem, and anger. The primary determinants of the effectiveness of education are: Observed behavior modification and compliance Self-reported behavior modification and compliance Positive reports from the probation officer Drug and alcohol abstinence verified by testing

Observed behavior modification and compliance Rationale: Two primary determinants of educational effectiveness include: Behavior modification, which involves thorough observation and measurement, identifying behavior that needs to be changed, and planning and instituting interventions to modify that behavior. Techniques include demonstrations of appropriate behavior, reinforcement, and monitoring until the new behavior is consistent. Compliance rates are often determined by observation, which should be done at intervals and on multiple occasions. In some cases, this may depend on self-reports or reports of others. If education is intended to improve individual health and reduce risk factors and this occurs, it is a good indication of compliance.

A 27-year-old male with a history of drug and alcohol abuse, stealing, and bullying behavior has been working with the psychiatric/mental health nurse on educational issues related to substance abuse, self- esteem, and anger. The primary determinants of the effectiveness of education are: Observed behavior modification and compliance Self-reported behavior modification and compliance Positive reports from the probation officer Drug and alcohol abstinence verified by testing

Observed behavior modification and compliance Rationale: Two primary determinants of educational effectiveness include: Behavior modification, which involves thorough observation and measurement, identifying behavior that needs to be changed, and planning and instituting interventions to modify that behavior. Techniques include demonstrations of appropriate behavior, reinforcement, and monitoring until the new behavior is consistent. Compliance rates are often determined by observation, which should be done at intervals and on multiple occasions. In some cases, this may depend on self-reports or reports of others. If education is intended to improve individual health and reduce risk factors and this occurs, it is a good indication of compliance.

The most important criterion for determining the degree of a patient's pain is: Physical indication, such as grimacing or guarding Moaning Patient report Patient history

Patient report Rationale: Patient report is the most important criterion for determining the degree of a patient's pain. People may perceive and express pain very differently, so unless drug-seeking or attention- seeking behavior can be established, the psychiatric/mental health nurse should accept the patient's degree of pain as reported. Some cultures encourage outward expressions of pain while others do not. Various pain scales may be used, depending on the age and cognitive ability of the patient. The most commonly used scale for adolescents and adults is the one-to-ten scale.

A patient newly diagnosed with bipolar disorder has been prescribed lithium. When teaching the patient about the medication, the initial symptoms that the nurse should advise the patient to expect include: Polydipsia, polyuria, and mild nausea Dysuria and constipation Headache, nausea, and vomiting Headache, dizziness, and tinnitus

Polydipsia, polyuria, and mild nausea Rationale: A patient newly diagnosed with bipolar disorder has been prescribed lithium. When teaching the patient about the medication, the initial symptoms that the nurse should advise the patient to expect include polydipsia, polyuria, and mild nausea. These symptoms should subside after two or three days, but the patient should be advised of the importance of drinking 2.5 to 3 liters of water daily to maintain fluid balance. The patient should also weigh him- or herself daily and check for signs of edema and sudden weight gain, which may indicate fluid retention.

Which of the following disorders is characterized by personality disintegration and distortion in the perception of reality, thought processes, and social development? Depression Bipolar disorder Schizophrenia Narcissistic personality disorder

Schizophrenia Rationale: Schizophrenia: Personality disintegration and distortion in the perception of reality, thought processes, and social development, including delusions, withdrawal, odd behavior, hallucinations, inability to care for self, disorganized speech, catatonia, alogia (inability to speak because of mental confusion or aphasia), hearing voices, and avolition. Depression: Depressed mood, profound and constant sense of hopelessness and despair, or loss of interest in all or almost all activities. Bipolar disorder: Mood swings that may include mania, depression, or both. Narcissistic personality disorder: Heightened feeling of self-importance, persistent patterns of grandiosity, a need for admiration, disregard for other people's rights, restraint in expression of feelings, and a lack of empathy.

Psychoeducation is of special importance to help manage symptoms and behavior for patients with: Schizophrenia spectrum disorder Anxiety Obsessive-compulsive disorder Depression

Schizophrenia spectrum disorder Rationale: Psychoeducation is of special importance to help manage symptoms and behavior for patients suffering from schizophrenia spectrum disorder and bipolar disorder: Schizophrenia: Topics include information about the disease, the effects of medications, and methods to test reality because some symptoms (such as voices or hallucinations) may persist.Bipolar disorder: Topics include patterns of disease, triggers, self-monitoring tools, medication management and adverse effects, and mood changes.

When completing the patient assessment and developing the plan of care with a patient with an eating disorder, it is especially important to ask the patient about: Motivation to change behavior Self-injurious behavior Sexual dysfunction Goal for weight

Self-injurious behavior Rationale: When completing the patient assessment and developing the plan of care with a patient with an eating disorder, it is especially important to ask the patient about self-injurious behavior. Patients with eating disorders often engage in superficial self-mutilating behaviors, such as cutting, burning, and hair pulling, and these actions may increase as an outlet for the patient's emotional distress as the eating disorder is controlled. All patients with eating disorders should be screened for self-injurious behavior and should be monitored carefully during therapy.

If a patient attempts suicide while hospitalized in the mental healthcare unit, this would be classified as a(n): Sentinel event Serious reportable event Negligent event Adverse event

Serious reportable event Rationale: If a patient attempts suicide while hospitalized in the mental healthcare unit, this would be classified as a serious reportable event (aka a "never event"). The National Quality Forum provides a list of serious reportable events in seven different categories: surgical or invasive procedures, product or device, patient protection (which includes suicide and attempted suicide), care management, environmental, radiologic, and potential criminal events.

When a psychiatric and mental health nurse says to a patient, "Tell me about how you were sexually abused by your father," this is an example of the non-therapeutic communication technique of: Interpreting Requesting an example Approving/disapproving Probing

Probing Rationale: When a psychiatric and mental health nurse says to a patient, "Tell me about how you were sexually abused by your father," this is an example of the non-therapeutic communication technique of probing. Probing usually involves pushing the patient to answer questions or give details about information the patient would prefer not to divulge or is not yet ready to divulge. Patients may react defensively and feel that the nurse values the patient only for the information and doesn't care about the patient's feelings.

A patient flirts with her male nurse and then complains to the administrative staff that the nurse is "coming on" to her. This is an example of: Sublimation Suppression Displacement Projection

Projection Rationale: Projection occurs when a patient attributes unacceptable personal feelings or behavior to someone else, such as a patient flirting with a nurse and then claiming the nurse was "coming on" to her. Sublimation occurs when a patient refocuses unacceptable drives, feelings, or impulses onto something acceptable. Suppression occurs when a patient voluntarily blocks or suppresses unpleasant feelings or memories from awareness. Displacement occurs when a patient transfers feelings from one person or thing to another that is less threatening.

A male patient has been following a female patient and claims that the female is "flirting" with him and using "sexual innuendos;" however, the female patient complains that the male patient is harassing and scaring her, and staff observations concur with the female patient's complaints. The male patient is most likely exhibiting: Introjection Projection Compensation Identification

Projection Rationale: If a male patient has been following a female patient and claims the female is "flirting" with him and using "sexual innuendoes" but the female patient complains that the male patient is harassing and scaring her, and staff observations concur with the female patient's complaints, then the male patient is most likely exhibiting projection, an ego defense mechanism in which the male patient is projecting his own feelings of attraction onto the female patient.

The primary purpose of the American Nurses Credentialing Center (ANCC) is to: Provide political support for nurses. Provide nursing education. Promote the career of nursing. Promote nursing excellence.

Promote nursing excellence. Rationale: The primary purpose of the American Nurses Credentialing Center (ANCC), a subsidiary of the ANA, is to provide nursing excellence and to improve health care both in the United States and internationally. The ANCC provides a number of different programs and services, including an accreditation program for nursing education, certificate programs for nurses to demonstrate expertise in various specialty areas, the Pathway to Excellence® program that recognizes organizations with a positive nursing environment, a knowledge center that provides educational materials, and the Magnet Recognition Program® that recognizes an institution's excellence in patient care.

The psychiatric/mental health nurse is meeting with 10 staff members to review quality improvement data. The most effective initial presentation method is to: Make copies of raw data and place in bound notebooks to give to staff. Provide a slide show presentation that summarizes the data of the report. Ask staff members what information they'd like to have and proceed from there. Read a summary of the report to the staff members

Provide a slide show presentation that summarizes the data of the report. Rationale: The initial presentation should include a summary of the data. A slide show or similar presentation is excellent because the information can be presented in graphs or charts. A presenter should always avoid reading a report verbatim, as this is often boring and hard to follow. Raw data should be available for those who are interested, but beginning with raw data may be confusing. Discussions are good following the report, but the purpose is to dispense information rather than determine what information people want.

The psychiatric and mental health nurse is teaching an unmotivated patient about managing her disorder and medications. The best approach is probably to: Provide short lessons and assignments. Use illustrated materials only. Institute a reward/punishment system. Provide independent study materials.

Provide short lessons and assignments. Rationale: If the psychiatric and mental health nurse is teaching an unmotivated patient about managing her disorder and medications, the best approach is probably to provide short lessons and assignments because lack of motivation often corresponds to short attention span. Illustrated materials may not always be appropriate, and the nurse should avoid punishing the patient, as this may further decrease her motivation to learn. Independent study materials require motivation to start with or the patient won't do the work needed.

If a therapist is basing therapy on the theory of behaviorism, the psychiatric and mental health nurse expects that the focus of the patient's care will be on: Providing negative reinforcement Providing positive reinforcement Providing unconditional positive regard Assessing the patient's needs

Providing positive reinforcement Rationale: If a therapist is basing therapy on the theory of behaviorism, the psychiatric and mental health nurse expects that the focus of the patient's care will be on providing positive reinforcement. Behaviorism is based on the theory that patients are essentially passive and behavior results from stimulus and response. Behaviors that are met with positive reinforcement are likely to be repeated. However, behaviors met with negative reinforcement are also likely to be repeated.

A patient deals with traumatic stress by accepting the stressful event, exposing herself to the stress by thinking about and describing the event, and working to understand the meaning of the event. This patient is engaged in: Repressive coping Reframing Rational coping Stress inoculation training

Rational coping Rationale: Rational coping requires the patient to actively engage a stressor even though it may be painful initially in order to move past the trauma. Rational coping comprises three steps: Acceptance: Accepting that the event happened rather than trying to deny or repress the event. Exposure: Exposing oneself to the stressor by thinking and talking about it and describing the event so that it loses power. Understanding: Making an effort to understand the meaning of the event in the person's life.

A patient who is abusive toward his son claims that the son was fathered by a different man with whom his wife was having an affair and is "incorrigible and lazy." The defense mechanism that the patient is using is: Compensation Suppression Denial Rationalization

Rationalization Rationale: If a patient who is abusive toward his son claims that the son was fathered by a different man with whom his wife was having an affair and is "incorrigible and lazy," the defense mechanism that the patient is using is rationalization. Rationalization is attempting to justify negative or unacceptable behavior by providing explanations that may seem reasonable to others rather than accepting blame.

According to Piaget, adulthood begins when the person is able to: Act independently. Develop a sense of morality. See the self as distinct from others. Reason systematically about abstract concepts

Reason systematically about abstract concepts Rationale: According to Piaget, adulthood begins when the person is able to reason systematically about abstract concepts. Piaget's 4 stages of cognitive development include: sensorimotor (birth to 2 years), preoperational (2 to 6 years), concrete operational (6 to 11 years), formal operational (11 through adulthood). Piaget believed that childhood ended with movement to the formal operational stage although Piaget recognized that not everyone completes the tasks at each stage of development and, thus, not everyone cognitively moves into adulthood.

During the orientation phase of building a therapeutic relationship, the psychiatric and mental health nurse discovers that he had come to the first meeting with preconceptions about the patient. Based on this, the nurse should: Ask another nurse to work with the patient. Apologize to the patient. Spend extra time with the patient. Recognize and set aside the preconceptions.

Recognize and set aside the preconceptions. Rationale: While ideally a nurse should examine preconceptions and set them aside prior to meeting with the patient, once the nurse recognizes that his opinions may be colored by preconceptions, he should acknowledge them and set them aside so that he can establish a good working relationship with the patient. Since the patient is likely unaware of the nurse's preconceptions, apologizing is not necessary, nor is overcompensating by spending extra time with the patient.

A patient may utilize the ego defense mechanism of sublimation in order to: Voluntarily block unpleasant emotions. Negate an intolerable experience Retreat to an earlier stage of development. Redirect socially unacceptable impulses

Redirect socially unacceptable impulses Rationale: Sublimation: The patient redirects socially unacceptable impulses to acceptable actions, such as when the victim of a crime redirects anger toward becoming an advocate for other victims. Regression: The patient retreats to an earlier stage of development, such as by being more dependent. Suppression: The patient voluntarily blocks unpleasant emotions, such as by refusing to think about an event. Repression: The patient involuntarily blocks unpleasant emotions, such as being unable to remember being raped.

A patient may utilize the ego defense mechanism of sublimation in order to: Voluntarily block unpleasant emotions. Negate an intolerable experience Retreat to an earlier stage of development. Redirect socially unacceptable impulses.

Redirect socially unacceptable impulses. Rationale: Sublimation: The patient redirects socially unacceptable impulses to acceptable actions, such as when the victim of a crime redirects anger toward becoming an advocate for other victims. Regression: The patient retreats to an earlier stage of development, such as by being more dependent. Suppression: The patient voluntarily blocks unpleasant emotions, such as by refusing to think about an event. Repression: The patient involuntarily blocks unpleasant emotions, such as being unable to remember being raped.

A patient tells the psychiatric and mental health nurse that he is "a total loser," and the nurse replies "You believe that you are a total loser." This is an example of: Reframing Negative reinforcement Repetition Reflection

Reflection Rationale: If a patient tells the psychiatric and mental health nurse that he is "a total loser," and the nurse replies "You believe that you are a total loser," this is an example of reflection. Reflection is repeating a patient's statements so that the patient hears the words and can perceive the meaning from a different perspective. Reflection holds up a mirror for the patient but also ensures that the patient knows that he or she has been heard and his or her concerns have been recognized.

When using the I-PASS mnemonic—(I) illness severity, (P) patient summary, (A) action list, (S) situation awareness and contingency planning, and (S) synthesis by receiver—to guide handoff, included in the patient summary is (are): The stability of the condition A plan for what may occur A timeline and responsibilities The events that led to the hospitalization

The events that led to the hospitalization Rationale: When using the I-PASS mnemonic to guide handoff, included in the patient summary are the events that led to hospitalization as well as the patient's course in the hospital (changes, problems, positive outcomes) and ongoing assessment of condition. The mnemonic is: (I) illness severity: acuity level. (P) patient summary: as above. (A) action list: interventions/actions, timeline. (S) situational awareness and contingency planning: what's happening and what may happen. (S) synthesis by receiver: summarize the information received.

A patient is receiving methylphenidate 10 mg BID for treatment of attention deficit hyperactivity disorder but has lost 10 pounds and is becoming underweight because of a loss of appetite. The psychiatric and mental health nurse should advise the patient to take: The medication upon arising and at bedtime The medication before meals Both doses of the medication at bedtime The medication after meals

The medication after meals

A patient who has had multiple arrests for driving under the influence of alcohol has agreed to begin treatment with disulfiram. Patient education should include advising the patient that: The patient may experience severe illness if they drink alcohol. The patient should avoid driving. The patient may experience hallucinations. The patient must abstain from drinking for one week prior to initiating treatment.

The patient may experience severe illness if they drink alcohol. Rationale: If a patient has agreed to begin treatment with disulfiram, the patient should be aware that drinking alcohol may result in severe illness. Patients must abstain from drinking for 12 hours before initiating treatment. Disulfiram interferes with the breakdown of acetaldehyde from ethanol, so the acetaldehyde level increases, resulting in a syndrome that can include flushing, head and neck pain, severe nausea and vomiting, third, excessive perspiration, tachycardia, hyperventilation, weakness, and blurred vision. Some people may develop more severe symptoms, such as myocardial infarction, acute heart failure, and/or respiratory depression.

The emphasis of reality therapy (Glasser) is on: The present and personal responsibility Past behavior and causes Labeling behavior and symptoms Stress reduction techniques

The present and personal responsibility Rationale: The emphasis of reality therapy (Glasser) is on the present, the here and now, and personal responsibility. Patients examine the ways in which their behavior has interfered with their ability to achieve goals. The view of reality therapy is that the individual is responsible for his or her choice of actions. According to this theory, people are born with 5 basic needs-power, belonging, freedom, fun, and survival—and people develop personality in trying to fulfill these needs. Present behavior is addressed, not past, unless behavior in the past has specifically affected present behavior.

What is the disengagement theory?

They theory that as people age, they begin to take themselves out of society and have responsibilities gradually taken away from them.

On physical examination, a patient with chronic alcohol use disorder exhibits ophthalmoplegia, ataxia, and confusion with stupor and somnolence. Based on these findings, the most likely cause is: Vitamin C deficiency Iron deficiency Thiamine deficiency Vitamin D deficiency

Thiamine deficiency (Vitamin B1 Deficiency) Rationale: If, on physical exam, a patient with chronic alcohol use disorder exhibits ophthalmoplegia, ataxia, and confusion with stupor and somnolence, the most likely cause is vitamin B deficiency, primarily thiamine (vitamin B1). These symptoms are the typical triad associated with Wernicke disease. Patients often also exhibit signs of Korsakoff's psychosis with anterograde and retrograde amnesia and confabulation, so this combination is referred to as Wernicke-Korsakoff syndrome. These conditions are life threatening if not treated aggressively with thiamine replacement.

If a schizophrenic patient believes that others know the thoughts in her mind, this delusional belief is called: Thought broadcasting Thought blocking Thought withdrawal Circumstantial thinking

Thought broadcasting Rationale: Thought broadcasting: The belief that one's thoughts can be heard or known by others. Thought blocking: Stopping in the middle of expressing an idea and being unable to regain the train of thought and continue and complete the statement. Thought withdrawal: The belief that one's thoughts are being taken away by someone else and that the individual cannot stop this process. Circumstantial thinking: Eventually responding to a question after providing excessive and unnecessary details.

A 25-year-old female with bipolar disorder is to begin treatment with lithium. Which laboratory tests should be carried out prior to beginning treatment with lithium? Thyroid function Liver function Renal function Cardiovascular function

Thyroid function Rationale: Before a patient begins treatment with lithium, thyroid function tests should be completed in order to ensure that hypothyroidism is not a contributing cause to the patient's depression and to serve as a baseline for subsequent monitoring of thyroid function. Lithium decreases production of thyroid hormones, so lithium-induced hypothyroidism can occur. If the baseline thyroid function tests are normal, then thyroid function is usually monitored every 6 to 12 months; but, if the TSH level is elevated, every 3 to 6 months. About 40 to 50% of patients receiving lithium develop goiter.

When assessing a patient's orientation, the psychiatric and mental health nurse should be aware that the first thing the patient is likely to lose track of is: Person Place Time Current situation

Time Rationale: When assessing a patient's orientation, the psychiatric and mental health nurse should be aware that the first thing the patient is likely to lose track of is time, followed by place and then person. Patients may, for example, forget the day of the week or the month and date. When orientation improves, it usually does so in the reverse order, so people become oriented to person first, followed by place, and then time

If a patient with psychosis divulges that he intends to kill his parents, healthcare providers must: Have the patient arrested. Warn the parents. Increase patient oversight. Advise the patient not to make threats.

Warn the patients Rationale: While what a patient says is usually protected by the regulations regarding privacy and confidentiality, if a patient makes a credible threat, such as intending to kill his parents, then the healthcare provider must warn the parents of the danger under the "duty to warn" laws. These laws may vary somewhat from one state to another with some states permitting healthcare providers to use professional judgment about warning others and other states requiring mandatory reporting.

Which of the following is a healthy response to conflict with another person? The belief that the other person's point of view is wrong Resentment toward the other person Willing to seek compromise with the other person Siding with the other person despite feelings of abandonment

Willing to seek compromise with the other person Rationale: A healthy response to conflict with another person is the ability to seek compromise and to let go of anger, disappointment, and resentment, which interfere with the healing process. Resolving conflicts is facilitated by a calm, reasonable approach that shows respect for the other individual despite the differences that serve as the basis of the conflict. The members to the conflict should make an effort to understand the feelings associated with the opinions.

A string of words that have absolutely no relation when speaking.

Word Salad

Which of the following is an example of secondary prevention for an at-risk adolescent? Refer to Alateen® if parents are alcoholics. Refer to a support group for children of divorce. Work with the patient to modify negative behavior. Provide sex-education courses.

Work with the patient to modify negative behavior. Rationale: Secondary prevention requires some type of intervention to deal with inappropriate behavior, such as by working with the patient to modify negative behavior. The adolescent patient may receive secondary prevention measures (treatment) in the community or as an inpatient. If the patient is hospitalized, the psychiatric and mental health nurse focuses on helping the patient learn more appropriate problem-solving skills and helping the patient (and family) stabilize crisis situations.

When conducting the physical examination on a patient, the psychiatric and mental health nurse notes that the patient has dysphonia and can only speak in a hoarse whisper, a symptom that has persisted for over 6 months. Based on this observation, the cranial nerve that should be assessed is: I (one) II (two) VIII (eight) X (ten)

X (ten) Rationale: Because the patient has longstanding dysphonia and can only speak in a hoarse whisper, the cranial nerve that should be assessed is cranial nerve X (ten), because it provides sensation and innervation to the larynx per the laryngeal nerves. Injury or paralysis of either or both of these nerves can result in persistent hoarseness. To assess cranial nerve X, the patient is asked to open the mouth and say "Ahh" while the nurse observes the movement of the soft palate and pharynx. Normal response is symmetrical elevation of the palate and bilateral medial movement of the pharynx with the uvula mid-center.

Rogers and Maslow (Humanistic Psychology)

doesn't believe in animal research - animal and humans are nothing alike and therefore they cannot give any information on the human experience - have optimistic view: humans can rise above, humans are conscious/rational beings and are not dominated by the subconscious

What is the abstinence violation effect?

feeling of loss of control that results when a person has violated self-imposed rules. Cognitive restructuring in which the lapse is distinguished from a relapse and seen an isolated mistake, one

Psychologists who carefully watch the behavior of chimpanzee societies in the jungle are using a research method known as

naturalistic observation

What is the activity theory?

the more active and involved older adults are, the more likely they are to be satisfied with their lives-both physically and mentally

A patient with schizophrenia with catatonia has sat in the same chair with the right arm extended for an hour after the phlebotomist extended the arm for a blood draw. This is an example of: posturing. waxy flexibility. anergia. mimicry.

waxy flexibility. Rationale: If a patient has sat in the same chair with the right arm extended for an hour after the phlebotomist extended the arm for a blood draw, this is an example of waxy flexibility, a psychomotor behavior associated with schizophrenia with catatonia. With waxy flexibility, the patient maintains a position initiated by someone else (such as the phlebotomist). This differs from posturing in that, with posturing, the patient voluntarily assumes abnormal or bizarre postures.

The psychiatric and mental health nurse has noted that, when the parents of an adolescent patient met with the psychiatrist to discuss the patient's condition and needs, they asked no questions but afterward seemed very confused about what the psychiatrist had said. The most appropriate approach is to say: "Can you tell me what you understand about what the doctor has told you?" "Did you understand what the doctor was telling you?" "Are you confused about what the doctor was telling you?" "Do you have any more questions?"

"Can you tell me what you understand about what the doctor has told you?" Rationale: If the psychiatric and mental health nurse has noted that, when the parents of an adolescent patient met with the psychiatrist to discuss the patient's condition and needs, they asked no questions but afterward seemed very confused about what the psychiatrist had said, the most appropriate approach is to say "Can you tell me what you understand about what the doctor has told you?" This allows the nurse to assess their level of understanding. Sometimes people are intimidated when speaking with healthcare providers, but the failure to ask questions and apparent confusion suggests that the patient may lack health literacy.

If a patient states he feels "life is pointless," an appropriate response is: "Everyone feels down at some time in his life." "Just be patient. You will feel better soon." "Why don't you try to think of some positive things in your life." "I can see you are upset. What are you feeling now?"

"I can see you are upset. What are you feeling now?" Rationale: If a patient states he feels "life is pointless," an appropriate response is "I can see you are upset. What are you feeling now?" because this acknowledges the patient's feelings and encourages the patient to explore the cause of the feelings without rejecting or belittling the patient's expressions. Statements that may be construed as suicidal should always be taken seriously and dealt with forthrightly, such as by asking the patient if he is considering suicide.

A patient experiencing auditory hallucinations tells that nurse that he hears voices warning him of danger: "Don't you hear them?" The best response is: "There are no voices. You are hallucinating." "I know the voices seem real to you, but I don't hear them." "I cannot make them out. What are they saying?" "Try to stay focused on what's real. There are no voices."

"I know the voices seem real to you, but I don't hear them." Rationale: The best response to a patient experiencing auditory hallucinations, saying "Don't you hear them?" is to state "I know the voices seem real to you, but I don't hear them." This response validates the patient's perception and real fear of the voices ("I know the voices seem real to you") while orienting the patient to reality ("I don't hear them"). The psychiatric and mental health nurse should speak in a calm voice and avoid standing too close to or touching the patient without permission, as these actions may increase the patient's fear and anxiety.

Which of the following statements by a patient indicates a readiness to learn? "I don't need to be hospitalized as there's nothing wrong with me.". "It's my mother's fault I ended up here." "I already know all I need to." "I need to be in better control of my life."

"I need to be in better control of my life." Rationale: "I need to be in better control of my life" is the statement that indicates a readiness to learn because the patient is expressing motivation. The four types of readiness to learn include (1) physical readiness (health, gender, vision, hearing), (2) emotional readiness (motivation, frame of mind, anxiety level, support system, developmental stage), (3) experiential readiness (cultural background, orientation, aspiration level, and (4) knowledge readiness (cognitive ability, learning style, learning disabilities, educational background).

During a meeting between a patient and the nurse, the patient paces back and forth and appears agitated and upset. Which of the following is the most appropriate response? "Why are you so upset?" "Take a moment and calm down before we proceed." "I notice you are pacing and seem upset." "Your pacing is making me uncomfortable."

"I notice you are pacing and seem upset." Rationale: The most appropriate response to a patient who is pacing and agitated is to describe the behavior observed: "I notice you are pacing and seem upset." The nurse should avoid appearing judgmental or asking "why" but should make the observation and allow the patient to respond, as this helps the patient to recognize behaviors. The nurse should also avoid being directive, such as with "Take a moment and calm down before we proceed," unless a patient's actions are endangering.

If a patient is using SMART goals as a framework, an appropriate specific goal statement is: "I will gain weight." "I will have a healthy attitude toward food." "I will gain a half pound or more weekly." "I will stop losing weight."

"I will gain a half pound or more weekly." Rationale: If a patient is using SMART goals as a framework, an appropriate specific goal statement is "I will gain a half pound or more weekly" because this is easily measurable. SMART stands for Specific: State concretely and in terms that can be measured.Measurable: Identify the model used to determine success or failure. Attainable/Achievable: Aim for a goal that is likely achievable. Realistic/Relevant: Avoid wishful thinking.Time-Limited/Trackable: Set a time frame to help the patient keep on track

This is the primitive, pleasure seeking mind that involves: --Food --Sex --Survival --Seeking immediate satisfaction

"Id"- Sigmund Freud

If a patient with schizophrenia spectrum disorder is having delusions and believes that the government is sending assassins to kill him or her, an appropriate response is: "It must be hard to be so scared, but you are safe here." "You are having a delusion. The government is not sending assassins." "Look around. This is a safe place, so you don't need to worry." "We have security guards that will protect you from the assassins."

"It must be hard to be so scared, but you are safe here." Rationale: If a patient with schizophrenia is having delusions and believes that the government is sending assassins to kill him, an appropriate response is "It must be hard to be so scared, but you are safe here." Although the patient is experiencing delusions, they are very real for the individual, and attempts to convince the person of reality will not be successful. It is important to show support by expressing empathy, but the nurse should not validate (by stating "We have security guards that will protect you from the assassins") or ridicule the delusion.

Which of the following statements by a psychiatric and mental health nurse demonstrates a good understanding of peer review? "I don't mind reviewing someone as long as my review is anonymous." "My peer review is going to get him fired for incompetence!" "Peer review is a good learning experience for me and the person I'm reviewing." "The supervisor should do the peer reviews because the supervisor has more authority."

"Peer review is a good learning experience for me and the person I'm reviewing." Rationale: The statement by a psychiatric and mental health nurse that demonstrates a good understanding of peer review is "Peer review is a good learning experience for me and the person I'm reviewing." The point of peer review is that the reviews are done by peers, those of the same rank, and not supervisors and never anonymously. The reviewer and the reviewee should discuss the review, with the reviewer prompting the reviewee to seek solutions to any problems that may have been identified.

If a family member of a patient asks the psychiatric and mental health nurse what constitutes probable cause for involuntary commitment, the best response is: "You should ask an attorney about that." "The person is a threat to herself or others." "The person is uncooperative with the family." "The person is no longer able to work and is homeless."

"The person is a threat to herself or others." Rationale: While laws may vary slightly from one state to another in relation to involuntary commitment, generally probable cause is present if a person is a threat to herself or others (and usually the threat must be imminent). A second criterion is usually that the person is too disabled to provide self-care; however, this last criterion can be interpreted in a wide variety of ways (the reason so many mentally ill individuals are homeless and living on the streets) and is rarely utilized.

During the initial interview, the patient states repeatedly that his boss is to blame for all of the patient's problems and that the boss "is going to pay." The psychiatric and mental health nurse should respond by asking: "Why do you feel that way?" "What thoughts have you had about hurting your boss?" "Can you think of other reasons for your problems?" "Do you think that this anger toward your boss is productive?"

"What thoughts have you had about hurting your boss?" Rationale: If, during an interview the patient blames his boss for his problems and states that the boss is "going to pay," this is an implied threat. Because of the duty to warn those who might be in danger from a patient with mental health issues, the psychiatric and mental health nurse should ask directly, "What thoughts have you had about hurting your boss?" in order to assess whether there is a risk of violence. In some cases, orientation may be extended to include the current situation of the patient.

Which of the following is an example of giving a broad opening as a therapeutic communication technique? "What seemed to lead up to your panic attack?" "What would you like to discuss this morning?" "I notice that you are wringing your hands." "I understand what you are saying."

"What would you like to discuss this morning?" Rationale: Therapeutic communication includes: Giving a broad opening: "What would you like to discuss this morning?" This allows the patient to control the interaction and shows respect for the individual. Establishing time sequence: "What seemed to lead up to your panic attack?" This helps to establish the relationship among different events. Observing: "I notice that you are wringing your hands." This helps the patient to recognize behaviors. Accepting: "I understand what you are saying." This helps to convey regard for the patient and reception for the patient's ideas.

Considering Maslow's hierarchy, in which order should the following nursing diagnoses for a patient be prioritized (first to last)? (1) Deficient fluid volume, (2) risk for self-injury, (3) sexual dysfunction, and (4) low self- esteem (1) Low self-esteem, (2) risk for self-injury, (3) low self-esteem, and (4) sexual dysfunction (1) Deficient fluid volume, (2) low self-esteem, (3) risk for self-injury, and (4) sexual dysfunction (1) Risk for self-injury, (2) deficient fluid volume, (3) sexual dysfunction, and (4) low self- esteem

(1) Deficient fluid volume, (2) risk for self-injury, (3) sexual dysfunction, and (4) low self- esteem Rationale: Considering Maslow's hierarchy, the order in which the nursing diagnoses for a patient should be prioritized (first to last) is: Physiological needs: Deficient fluid volume. Safety needs: Risk for self-injury.Love/belonging needs: Sexual dysfunction. Esteem needs: Low self-esteem. Physiological needs, especially those that are critical to life, should always be a top priority. However, prioritizing does not necessarily mean that the first priority must be dealt with before the psychiatric and mental health nurse can deal with the second priority because, in reality, many diagnoses may be attended to simultaneously.

Maslow's Hierarchy of Needs

(level 1) Physiological Needs, (level 2) Safety and Security, (level 3) Relationships, Love and Affection, (level 4) Self Esteem, (level 5) Self Actualization

The term that Latinos may use to describe someone who is vulnerable to the stresses of life is: Tonto Nervios Enfermo Malo

Nervios Rationale: Nervios is the term that Latinos may use to describe someone who is vulnerable to the stresses of life. Other terms that Latinos may use include: Susto: Condition caused by a frightening experience or event.Locura: Condition in which the person exhibits severe psychosis.Bilis, cólera: Condition caused by suppressing anger/rage that causes core imbalance.

What is the antidote for toxic alcohols?

10% ethanol infusion and/or dialysis

The patient that would likely derive the most benefit from Assertive Community Treatment (ACT) is a: 65-year-old male with history of liver disease and severe alcohol use disorder. 40-year-old male with history of history of severe schizophrenia and alcohol use disorder. 30-year-old female recovering from injuries related to intimate-partner abuse. 20-year-old male recovering from methamphetamine use disorder.

40-year-old male with history of history of severe schizophrenia and alcohol use disorder. Rationale: The patient that would likely derive the most benefit from Assertive Community Treatment (ACT) is a 40-year-old male with history of history of severe schizophrenia and alcohol use disorder. ACT is designed to treat patient with severe and complex multiple health problems. A case manager is part of a team of members with specialties in psychiatry, social work, nursing, vocational rehabilitation, and substance abuse with services provided 24 hours a day every day of the year in order to lesson symptoms, meet the patients' needs, lesson the families' burdens, and promote independence.

In research, an example of a special-cause quality variance is: Procedures that are not clearly outlined A project that takes longer than anticipated Poor working conditions that affect outcomes A computer that crashes and data are lost

A computer that crashes and data are lost Rationale: A special-cause quality variance is one that is unexpected and cannot be anticipated, such as when a computer crashes and data are lost. When measuring data, some variance is expected, and these variances may be common cause or special cause. Common-cause variances are those that are to some degree predictable (such as poor working conditions affecting outcomes, procedures not clearly outlined, and projects taking longer than anticipated) and are the result of the system itself. Common-cause variances are of less concern than special-cause variances.

delirium tremens (DTs)

A severe withdrawal syndrom seen in alcoholics who are deprived of ethyl alcohol; characterized by restlessness, fever, sweating, disorientation, agitation, and seizures; can be fatal if untreated.

A 22-year-old female is receiving haloperidol for schizoaffective disorder. She was admitted to the psychiatric unit with delusional thinking, rapid disorganized speech, irritability, and lethargy. She has begun slapping at her face repeatedly. Which assessment tool is most indicated? Abnormal Involuntary Movement Scale (AIMS) CAGE Mini-mental state exam (MMSE) Confusion Assessment Method (CAM)

Abnormal Involuntary Movement Scale (AIMS) Rationale: AIMS is an assessment tool to evaluate tardive dyskinesia in those taking antipsychotic medications such as haloperidol. The CAGE tool is used as a quick assessment tool to determine if people are drinking excessively or are problem drinkers. MMSE is used to assess cognition in those with evidence of dementia or short-term memory loss associated with Alzheimer's disease. CAM is used to assess the development of delirium and is intended primarily for those without psychiatric training.

According to Havighurst's theory of adult development, middle age is characterized by: Managing a home and finding a congenial social group Establishing ties with those in the same age group and adjusting to a decrease in physical strength Establishing physical living arrangements that are satisfactory Achieving civil/social responsibility and maintaining an economic standard of living

Achieving civil/social responsibility and maintaining an economic standard of living Rationale: Havighurst's middle age tasks include --achieving civic/social responsibility, --maintaining an economic standard of living, --raising teenagers and teaching them to be responsible adults, --developing leisure activity, --accepting physiological changes related to aging, --adjusting to aging of parents. Early adulthood tasks include --finding a mate, marrying, having children, -- managing a home, --getting started in an occupation/profession, --assuming civic responsibility, and --finding a congenial social group. Older adulthood tasks include --adjusting to a decrease in physical strength and health, --death of a spouse, --life in retirement and reduced income, --establishing ties with those in the same age group, --meeting social and civic obligations, --establishing satisfactory physical living arrangements.

The four nonverbal behaviors that are associated with active listening include sitting: Beside the patient, maintaining open posture, leaning back comfortably, and maintaining eye contact Across from patient, maintaining closed posture, leaning forward, and avoiding eye contact Across from patient, maintaining open posture, leaning forward, and maintaining eye contact Beside the patient, maintaining open posture, leaning forward and maintaining eye contact

Across from patient, maintaining closed posture, leaning forward, and avoiding eye contact Rationale: The four nonverbal behaviors associated with active listening include: Sit across from patient: Facing the patient directly helps to convey interest. Maintain open posture: Keeping the arms and legs uncrossed helps to show the person is open to the other person's ideas and is less defensive than a closed position. Lean forward: Leaning toward the patient slightly shows engagement in the interaction. Maintain eye contact: Maintaining eye contact helps to show interest in the person; however, the psychiatric and mental health nurse should keep cultural differences in mind as direct eye contact is not the norm in all cultures

Under the NANDA-I Approved Nursing Diagnoses categories, "anxiety" is an example of which type of diagnosis? Risk Health promotion Actual Syndrome

Actual

The Peer-to-Peer program of the National Alliance on Mental Illness (NAMI) focuses on providing classes for: Family and caregivers of a children and adolescents with mental health conditions Families, partners, and friends of adults with mental illness Families, partners, and friends of military service members or veterans Adults with mental illness about mental illness

Adults with mental illness about mental illness Rationale: While most other programs of the National Alliance on Mental Illness (NAMI) provide classes to support family, partners, and friends of patients with mental illness or to educate mental health staff, the Peer-to-Peer program is aimed at people with mental illness. Peer-to-Peer provides 10 sessions of education about dealing with mental illness to assist those who want guidance in working toward recovery and to help people develop their own relapse prevention programs and learn to better interact with healthcare providers.

The type of exercise that may reduce symptoms of anxiety and depression is: Aerobic Isometric Stretching Weight lifting

Aerobic Rationale: The type of exercise that may reduce symptoms of anxiety and depression is aerobic exercise, such as jogging, walking, gardening, and riding bicycles. Psychiatric and mental health patients often lead relatively sedentary lives because of their symptoms and adverse effects associated with their medications, increasing their risks for obesity, diabetes, and heart disease. An exercise regimen should be part of lifestyle changes and should include a minimum of 30 minutes of aerobic exercise at least 3 times weekly.

Screening for intimate partner abuse should be done for all female patients: Age 12 or older Age 14 or older Age 16 or older Age 18 or older

Age 14 or older Rationale: While most screening for intimate partner abuse focuses on females age 18 or older, all female patients ages 14 and older should be screened, as many very young females engage in sexual relationships, and this makes them vulnerable to intimate partner abuse. Additionally, some screening may also be appropriate for males, who are sometimes also the victims of abuse. Intimate partner abuse also occurs in same sex relationships, both male and female, so healthcare providers should be alert to those possibilities.

When describing extrapyramidal effects of drugs, which of the following terms refers to the inability to start a movement? Akinesia Akathisia Dystonia Bradykinesia

Akinesia Rationale: Akinesia is the inability to start a movement. The extrapyramidal system is a group of neural connections outside of the medulla that control movement. Extrapyramidal effects are the result of drug influence on the extrapyramidal system and also include: Akathisia (inability to stop movement).Dystonia (extreme and uncontrolled muscle contraction, torticollis, flexing, and twisting). Bradykinesia (slowness of movement; also characteristic of Parkinson's disease). Tardive dyskinesia (inability to control movement such as tics, lip smacking, and eye blinking).

Two staff nurses in the psychiatric unit disagree about the best way to carry out duties, resulting in ongoing conflict and refusal to work together. The first step in resolving this conflict is to: Allow both individuals to present their side of the conflict without bias. Encourage them to reach a compromise. Tell them they are violating professional standards of conduct. Make a decision about the matter.

Allow both individuals to present their side of the conflict without bias. Rationale: Steps to conflict resolution include: First, allow both sides to present their side of conflict without bias, maintaining a focus on opinions rather than individuals. Encourage cooperation through negotiation and compromise. Maintain the focus, providing guidance to keep the discussions on track and avoid arguments. Evaluate the need for renegotiation, formal resolution process, or third party. The best time for conflict resolution is when differences emerge but before open conflict and hardening of positions occur. It is beneficial to pay close attention to the people and problems involved, listen carefully, and reassure those involved that their points of view are understood.

According to Bleuler, the "four A's" that are fundamental to thought disorders include affect, autistic thinking, loosening of associations, and: Addiction Abuse Ambivalence Acute anxiety

Ambivalence Rationale: According to Eugen Bleuler, the "four A's" that are fundamental to thought disorders include: Affect: Flat, blunted, inappropriate, labile Ambivalence: Having two different ideas or emotions simultaneously Autistic thinking: Private fantasy world causing abnormal responses to others; dereism (illogical thoughts not rooted in reality) Loosening of associations: Rapidly changing from one topic to another

A patient who has not responded well to other antipsychotics is started on quetiapine. In the initial period when the patient's medication dosage is being adjusted, the patient should be assisted with: Ambulation Toileting Dressing Eating

Ambulation Rationale: In the initial period when the patient's medication dosage is being adjusted, the patient should be assisted with ambulation because the patient is at increased risk of falls because of the hypotensive effects of the drug. The patient should be cautioned to sit up slowly and to stay seated for a moment before standing to reduce orthostatic hypotension. The patient's white blood cell count should also be monitored, as this drug may cause leukopenia as well as increased glucose levels and liver enzymes.

The organization that provides a wide range of continuing education courses, webinars, and podcasts regarding psychiatric mental health nursing is: American Nurses Credentialing Center (ANCC) National Alliance on Mental Illness (NAMI) American Psychiatric Nurses Association (APNA) American Psychiatric Nurses Association (APNA)

American Psychiatric Nurses Association (APNA) Rationale: The organization that provides a wide range of continuing education courses, webinars, and podcasts regarding psychiatric mental health nursing is the American Psychiatric Nurses Association (APNA). Both members and non-members can browse lists of continuing education resources in the eLearning Center although some courses are restricted to APNA members. Additionally, the APNA sponsors two conferences annually. The APNA also advocates for mental health care and represents more than 9000 nurses psychiatric and mental health nurses.

The organization that provides a wide range of continuing education courses, webinars, and podcasts regarding psychiatric mental health nursing is the: American Nurses Credentialing Center (ANCC) National Alliance on Mental Illness (NAMI) American Psychiatric Nurses Association (APNA) American Psychiatric Nurses Association (APNA)

American Psychiatric Nurses Association (APNA) Rationale: The organization that provides a wide range of continuing education courses, webinars, and podcasts regarding psychiatric mental health nursing is the American Psychiatric Nurses Association (APNA). Both members and non-members can browse lists of continuing education resources in the eLearning Center although some courses are restricted to APNA members. Additionally, the APNA sponsors two conferences annually. The APNA also advocates for mental health care and represents more than 9000 nurses psychiatric and mental health nurses.

A patient experienced a severe emotional crisis after her only child left home for college. This type of crisis is classified as: Maturational/developmental Dispositional Traumatic stress An anticipated life transition

An anticipated life transition Rationale: A patient experiencing a severe emotional crisis after her only child leaves home for college is undergoing a crisis of anticipated life transition, which involves a situation that the patient can anticipate and plan for but over which the patient may feel powerless and out of control. Other types of crises include maturational/developmental, dispositional, traumatic stress-related, and psychopathology-related. Patients experiencing a crisis are often not able to problem solve and need support and assistance in order to resolve the crisis.

If a patient who has an advance directive stating specifically that the patient does not want to be resuscitated attempts suicide by hanging and is found by a family member but is nonresponsive after being cut down, the correct action is to: Allow patient to die. Attempt to resuscitate the patient. Attempt resuscitation while contacting legal counsel. Ask family member for guidance regarding resuscitation.

Attempt to resuscitate the patient. Rationale: If a patient who has an advance directive stating specifically that the patient does not want to be resuscitated attempts suicide by hanging and is found by a family member but is nonresponsive after being cut down, the correct action is to attempt resuscitation. While people have the right to state their preference for no resuscitation, in most states this directive is not legally binding. Additionally, the do-not-resuscitate directive was never intended to facilitate suicide.

As part of milieu therapy, the psychiatric and mental health nurse should expect to: Provide weekly patient feedback. Attend regular community meetings. Direct patient participation. Establish rules of patient behavior.

Attend regular community meetings Rationale: With milieu therapy, all aspects of the patient's environment are considered therapeutic, and patients are expected to be active participants in planning their own treatment. Criteria for milieu therapy include fulfilling basic physiological needs, establishing an environment that is conducive to achieving therapeutic goals, and creating a democratic form of governance in which the patients and staff are equal members and participants. Patients and staff meet in weekly community meetings in order to establish rules, norms, and behavioral expectations and limitations.

Following the death of her infant daughter, a patient suddenly started attending church and praying obsessively while neglecting her husband and other children. According to Kübler-Ross's stages of grief, the patient is probably in what stage? Denial Anger Depression Bargaining

Bargaining Rationale: The patient is probably in the stage of bargaining, which is often characterized by increased religious practice, such as praying, as a way to "bargain" with God to help the person cope or to somehow (even magically) change the outcome. Stages include: Stage 1: Denial Stage 2: Anger Stage 3: Bargaining Stage 4: Depression Stage 5: Acceptance

If discussing assertiveness with a patient, the psychiatric and mental health nurse suggests a strategy in which the patient agree with a critical person without becoming defensive or agreeing to make a behavior change. For example, if a patient's employer says "You are always behind in your deadlines," the person's response is "Yes, I have been behind in two deadlines this year." This is an example of: Defusing Shifting from content to process Clouding/fogging Responding assertively with irony

Clouding/fogging Rationale: If a patient's employer says "You are always behind in your deadlines," and the person's response is "Yes, I have been behind in two deadlines this year," this is an example of clouding/fogging. The purpose is to answer without becoming defensive while stating what is true but without making a commitment to changing behavior, especially if the expected change is unreasonable. This type of response is often employed to defuse the situation when someone is being aggressive or manipulative.

The substance of abuse that may result in the development of movement disorders, such as Parkinson's disease, is: Alcohol Methamphetamine Cocaine Phencyclidine

Cocaine Rationale: The substance of abuse that may result in the development of movement disorders, such as Parkinson's disease, is cocaine. The long-term effects of cocaine may vary according to the method used for self-administration. For example, snorting cocaine may result in erosion of the nasal septum. Injecting cocaine intravenously is typically mostly associated with movement disorders. When reporting cocaine use, patients may use street names for the drug, such as coke, Charlie, a bump, snow, nose candy, sniff, lines, and blow.

The treatment that is most often indicated for patients with hoarding disorder is: Dialectical behavior therapy Transference-focused psychotherapy Psychoanalysis Cognitive behavioral therapy (CBT)

Cognitive behavior therapy (CBT) Rationale: The treatment that is most often indicated for patients with hoarding disorder is cognitive behavioral therapy (CBT), which helps patients recognize cognitive distortions and engage in cognitive reframing. Hoarding disorder is related to obsessive-compulsive disorder and is characterized by the inability to discard items, even those of no value, and it may include excessive acquisition, such as by purchasing multiples of the same item or by compulsive shopping.

In Piaget's theory of cognitive development, the stage in which the child or adolescent has a better understanding of cause and effect and a good ability to understand concrete objects and the concept of conservation is: Concrete operational Preoperational Sensorimotor Formal operational

Concrete operational Rationale: According to Piaget's theory of cognitive development, in the concrete operational stage, cause and effect is better understood and children have a good ability to understand concrete objects and the concept of conservation. Stages include: Sensorimotor (0-24 months): Intellect begins to develop and children acquire motor and reasoning skills, begin to use language, and prepare for more complex intellectual activities. Preoperational (2-7 years): Children develop a beginning concept of cause and effect along with magical thinking and egocentrism. Concrete operational (7-11 years): Children understand cause and effect, concrete objects, and the concept of conservation. Formal operational (11 years-adult): Children/young adults develop mature though processes, ability to think abstractly, and evaluate different possibilities and outcomes.

An older adult with a urinary infection may exhibit: Confusion Hallucinations Depression Anxiety

Confusion Rationale: An older adult with a urinary infection may exhibit confusion rather than the more typical symptoms of burning and frequency experienced by younger adults, so urinary tract infection should be suspected in an older adult who has sudden onset of confusion or sudden worsening of pre-existing dementia. Confusion is more likely to occur with severe infections that have spread to the kidneys. The confusion associated with urinary tract infection usually clears rapidly once the infection is treated.

The most appropriate tool to differentiate delirium from other types of confusion is the: Mini-Mental State Exam Confusion Assessment Method Mini-cog Time and Change Test

Confusion Assessment Method Rationale: The most appropriate tool to differentiate delirium from other types of confusion is the Confusion Assessment Method, which can be used for those without psychiatric training. The tool covers 9 different factors associated with abnormal behavior: onset, attention, thinking, level of consciousness, orientation, memory, perceptual disturbances, psychomotor abnormalities, and sleep-wake cycle. An acute onset of confusion with fluctuation in attention level and disorganized thoughts or an altered state of consciousness is characteristic of delirium.

If a patient shows signs of delirium, the assessment tool that can be used to differentiate delirium from other causes of confusion is the: Confusion Assessment Method Abnormal Involuntary Movement Scale Mini-Cog Mini-Mental State Examination

Confusion Assessment Method Rationale: If a patient shows signs of delirium, the assessment tool that can be used to differentiate delirium from other causes of confusion is the Confusion Assessment Method. The Abnormal Involuntary Movement Scale is used to assess tardive dyskinesia in patients taking antipsychotics. The Mini-Cog and the Mini-Mental State Examination are used to diagnose and plot the course of dementia.

A patient who has developed sudden onset of blindness with no identifiable physical cause seems completely unconcerned about the deficit. This suggests: Somatization disorder Pain disorder Conversion disorder Body dysmorphic disorder

Conversion disorder Rationale: Conversion disorder: Sudden onset of sensory (seeing, hearing) or motor (paralysis, weakness) deficits without identifiable physical cause. La belle indifference (unconcern) is common. Somatization disorder: Combinations of multiple physical symptoms, usually involving pain and sexual, gastrointestinal and/or pseudoneurological symptoms. Pain disorder: Pain that is unrelieved by analgesia and is affected by psychological status. Body dysmorphic disorder: Preoccupation with imagined physical defect or exaggeration of a physical defect, such as belief that one's nose is hideous, and often seeking surgical correction.

Suicide attempts during hospitalization have resulted in an average of 7 extra days of hospitalization, with extra cost of approximately $14,000 for each attempt. Various interventions are being considered. The method used to determine monetary savings resulting from planned interventions is the: Cost-benefit analysis Cost-effective analysis Efficacy study Cost-utility analysis

Cost-benefit analysis Rationale: Cost-benefit analysis uses average cost of an event and the cost of intervention to demonstrate savings. Cost-effective analysis measures the effectiveness of an intervention rather than the monetary savings. Efficacy studies may compare a series of cost-benefit analyses to determine the intervention with the best cost benefit. They may also be used for process or product evaluation. Cost-utility analysis is essentially a sub-type of cost-effective analysis, but it is more complex and the results are more difficult to quantify and use to justify expense because cost- utility analysis measures benefit to society in general, such as decreasing teen pregnancy.

When a psychiatric and mental health nurse has difficulty setting limits on a patient's behavior because of an emotional response to a patient, this is an example of: Transference Counter transference Magnification Codependency

Counter transference Rationale: When a psychiatric and mental health nurse has difficulty setting limits on a patient's behavior because of an emotional response to a patient, this is an example of counter transference, which refers to the emotional response the psychiatric and mental health nurse has toward the patient. Transference occurs when the patient transfers feelings and behavior that had been directed toward another person toward the nurse. For example, if the patient hates his mother and the nurse makes a comment that reminds him of her, the patient may transfer his negative feelings toward the nurse.

A psychiatric and mental health nurse finds herself feeling very angry toward a patient whose physical appearance and manner remind her of her abusive father. This is an example of which of the following? Countertransference Transference Displacement Projection

Countertransference Rationale: If a psychiatric and mental health nurse finds herself feeling angry toward a patient whose physical appearance and manner remind her of her abusive father, this is an example of countertransference because the nurse is displacing feelings toward her father onto the patient. It's important to recognize countertransference and to examine the cause in order to increase self-awareness. In some cases, the nurse may need to discuss the issue with colleagues. Transference occurs when the patient displaces feelings for others onto the nurse.

Patients with paraphilias often come into therapy as a result of: Desire for change Co-morbidity with serious psychiatric disorders Family pressure Criminal prosecution

Criminal prosecution Rationale: Patients with paraphilias often come into therapy as the result of criminal prosecution related to the activity. Most people with paraphilias do not desire to change the behavior, and most are very secretive about the practices. Common elements of paraphilias include sexual fantasies or arousal and sexual intercourse related to non-human or non-living objects, children, or non- consenting adults. Arousal often results from suffering or humiliation of the victim. Paraphilias usually start after puberty and persist throughout life, often resulting in significant social and occupational impairment.

Which of the following actions is most likely to be effective in establishing a relationship of trust with a newly admitted patient? Administering patient medications Providing the patient a list of unit rules Explaining the reason for unit procedures Giving the patient a tour of the facility

Explaining the reason for unit procedures Rationale: The action that is most likely to be effective in establishing a relationship of trust with a newly admitted patient is explaining the reason for unit procedures. While routine care—administering medications, providing a list of rules, and giving a tour—may be done in such a caring manner as to engender trust, those things that are "extra," such as taking time to explain procedures, keeping one's word, providing needed food or supplies, considering the patient's preferences, and ensuring confidentiality, are more likely to cause the patient to have confidence and trust in the psychiatric and mental health nurse.

Therapy for obsessive-compulsive disorder (OCD) usually includes: Psychodynamic psychotherapy Flooding Meditation Exposure and response prevention (ERP)

Exposure and response prevention (ERP) Rationale: Therapy for obsessive-compulsive disorder (OCD) usually includes exposure and response prevention (ERP), a specific component of cognitive behavioral therapy (CBT) designed to help patients with OCD lesson or extinguish compulsive responses. Patients rank order stressors and then, in a systematic manner, are exposed to triggers while trying not to respond with ritualistic behavior. Over time, patients should be able to face triggers without responding, but compliance with therapy is relatively poor. Other aspects of CBT are also included in therapy, and some benefit from meditation. Psychodynamic psychotherapy does not generally help relieve OCD symptoms.

What are the four principles of Motivational Interviewing?

Express empathy Develop discrepancy Roll with resistance Support self-efficacy

A common adverse effect of the extended use of antipsychotics, such as clozapine or risperidone, is: Nausea and vomiting Urinary retention Extrapyramidal symptoms (EPS) Abdominal cramping and diarrhea

Extrapyramidal symptoms (EPS) Rationale: A common adverse effect of the extended use of antipsychotics, such as clozapine or risperidone, is extrapyramidal symptoms (EPS), which may include bradykinesia, tardive dyskinesia, dystonia (muscle contractions and involuntary movements), akathisia (restless movements), and parkinsonism (tremor, shuffling gait, slow movement). Clozapine is the least likely to cause EPS, and risperidone is the most likely. Patients are often prescribed another medication, such as a benztropine, to help control EPS.

Which theory states that a change in one family member's behavior will affect others in the family? Health belief model (Rosenstock) Theory of planned behavior (Ajzen) Family systems theory (Bowen) Theory of reasoned action (Fishbein and Ajzen)

Family systems theory (Bowen) Rationale: Family systems theory states that members of a family have different roles and behavioral patterns, so a change in one person's behavior will affect the others in the family. The health belief model predicts health behavior with the understanding that people take a health action to avoid negative consequences if the person expects that the negative outcome can be avoided and that he/she is able to do the action. The theory of reasoned action states the actions people take voluntarily can be predicted according to their personal attitude toward the action and their perception of how others will view the action they do. The theory of planned behavior evolved from the theory of reasoned action when studies showed behavioral intention does not necessarily result in action.

A 16-year-old male admitted to the mental health unit for alcohol use disorder has repeatedly failed to maintain sobriety and consistently missed support meetings while partying with his friends. What is the most likely reason that the patient is not compliant with treatment? Disturbance of body image Embarrassment Fear of being different from peers Guilt about illness

Fear of being different from peers Rationale: A 16-year-old patient who has repeatedly failed to maintain sobriety and consistently missed support meetings while partying with his friends has most likely done so out of fear of being different from his peers. Peer relationships are especially important to adolescents who are still developing a sense of self, so if an adolescent is involved in drinking with his friends, he may be reluctant to change the dynamic by remaining sober and may feel he will be abandoned or ridiculed if his behavior changes.

The purpose of the Life Safety Code® is: Disease prevention Accident prevention Infection control Fire prevention

Fire prevention Rationale: The Life Safety Code® (NFPA 101) is produced by the National Fire Protection Association in order to prevent fires. The Life Safety Code® is a set of standards rather than laws or regulations, but the standards are widely followed and laws usually conform to the standards. The Life Safety Code® focuses on fire-safe construction for various types of facilities, including hospitals, and applies to new construction as well as existing structures, vehicles, and vessels. The Life Safety Code® ranks the flame spread of materials.

When determining the burden of proof for acts of negligence, risk management would classify willfully providing inadequate care while disregarding the safety and security of another as: Negligent conduct Gross negligence Contributory negligence Comparative negligence

Gross negligence Rationale: Gross negligence. Negligence indicates that proper care has not been provided, based on established standards. Reasonable care uses a rationale for decision-making in relation to providing care. Types of negligence include: Negligent conduct (indicates that an individual failed to provide reasonable care or to protect/assist another, based on standards and expertise). Gross negligence (willfully providing inadequate care while disregarding the safety and security of another). Contributory negligence (involves the injured party contributing to his or her own harm). Comparative negligence (attempts to determine what percentage amount of negligence is attributed to each individual involved).

The most common behavioral therapy used to help patients with Tourette's syndrome control tics is: Interoceptive exposure Contingency management Massed negative practice Habit reversal training.

Habit reversal training Rationale: The most common behavioral therapy used to help patients with Tourette's syndrome control tics is habit reversal training, which helps patients recognize habitual pattern and motor sequences associated with tics so they can identify the times and conditions under which the urge to tic occurs. People learn routines to counter the tic, such as breathing slowly with the mouth closed to control vocal tics or covering the mouth as for a cough with a tic that involves sticking out the tongue

If a patient with major depressive disorder is to receive an SSRI that has been associated with long QT syndrome and tachycardia characterized by torsade de pointes, the patient should: Be advised to notify the MD if any symptoms develop. Have a baseline ECG and periodic follow-up ECGs. Be administered the lowest possible dose of the medication. Receive a beta-blocker in addition to the medication.

Have a baseline ECG and periodic follow-up ECGs. Rationale: If a patient is to receive an SSRI for depression that has been associated with long QT syndrome (LQTS) and tachycardia characterized by torsade de pointes, the patient should have a baseline ECG to evaluate for congenital LQTS, periodic ECGs after therapy is initiated, and a careful personal history/medical history, and detailed family history completed, with special attention to cardiac abnormalities. LQTS may be induced by dozens of drugs, including SSRIs and antipsychotic medications, especially in patients with preexisting congenital abnormalities.

A realistic goal for relapse prevention therapy (RPT) is to: Prevent all relapses. Emphasize the negative aspects of relapse. End the patient's association with previous friends. Help patients deal with the potential for relapse.

Help patients deal with the potential for relapse. Rationale: A realistic goal of relapse prevention therapy (RPT) is to help patients deal with the potential for relapse. RPT is sometimes used as the primary substance abuse treatment but may also be used for aftercare to encourage compliance. The program uses cognitive (reframing) and behavioral (meditation, exercise) techniques to teach patients to understand the process of relapse, identify situations that place them at risk of relapse, cope with cravings, recover after relapse and continue the program, and learn to lead a more satisfying and balanced life.

A group cognitive behavioral therapy (CBT) approach that focuses on relapse prevention for substance use disorders will likely: Stress the importance of attending Alcoholics or Narcotics Anonymous® (AA) meetings. Stress mindfulness and accepting oneself. Help patients identify situations that make them vulnerable to relapse. Advise patients to serve as mentors for each other.

Help patients identify situations that make them vulnerable to relapse. Rationale: A cognitive behavioral therapy (CBT) approach that focuses on relapse prevention for drug use disorders will likely help patients identify situations that make them vulnerable to relapse. Therapy may include training in behavioral skills and the use of cognitive interventions to assist them to identify triggers or situations that result in relapse as well as to provide tools they can use if faced with a situation that is placing the patient at risk, such as when associates are engaging in addictive behavior.

If the patient is in the precontemplation stage of change regarding smoking, according to the Transtheoretical Model (TTM), the initial step in helping the patient quit smoking through a self-help program should be to: Advise the patient to wait until the patient is psychologically ready. Advise the patient to immediately begin the self-help program. Advise the patient that self-help programs are generally ineffective. Help the patient progress beyond the stage of precontemplation.

Help the patient progress beyond the stage of precontemplation. Rationale: If the patient is in the precontemplation stage of change regarding smoking, according to the Transtheoretical Model (TTM), the initial step in helping the patient quit smoking through a self- help program should be to help the patient progress beyond the state of precontemplation with a brief intervention, which may include educating the patient and helping motivate the patient to change. Studies have shown that failure rates are high if patients attempt change from a baseline precontemplation stage (92%) with the failure rate decreasing if the patient begins at Contemplation (85%) or Preparation (75%).

Which physiological response is common when patients are undergoing acute withdrawal from opioids? Hypertension Hypotension Somnolence Constipation

Hypertension Rationale: Patients undergoing acute withdrawal from opioids typically exhibit hypertension and tachycardia as well as piloerection, diaphoresis, rhinorrhea, diarrhea, muscle cramps, arthralgia, nausea, vomiting, anxiety, and lethargy. The peak period for symptoms is one to three days after cessation with symptoms lasing for five to seven days. Medications used for treatment of opioid withdrawal include clonidine or methadone. Naltrexone is sometimes used to block opioid receptors so that the drug does not produce euphoria, but the patient must be without symptoms of withdrawal for at least a week before naltrexone is started.

A patient with schizophrenia is admitted to the hospital after developing polydipsia, drinking water excessively and becoming increasing confused and psychotic. The electrolyte imbalance of most concern is: Hyponatremia Hypernatremia Hypocalcemia Hypercalcemia

Hyponatremia Rationale: The electrolyte imbalance of most concern with polydipsia, which is characterized by excessively drinking water >3L per day, is hyponatremia because of the diluting effect that the water has on the blood and the inability of the kidneys to excrete urine fast enough. Polydipsia may occur with schizophrenia as well as in those with developmental disabilities. If untreated, the patient may develop seizures and experience cardiac arrest. Treatment of polydipsia generally requires hospitalization. Clozapine is often used to control symptoms.

According to Erikson's psychosocial theory and stages of development, a 30-year-old male who remains very insecure and dependent on his parents and still lives at home has probably not successfully achieved the stage of: Trust vs mistrust Identity vs role confusion Industry vs inferiority Initiative vs guilt

Identity vs role confusion Rationale: According to Erikson's psychosocial theory and stages of development, a 30-year-old male who remains very insecure and dependent on his parents and still lives at home has probably not successfully achieved the stage of identity vs role confusion, which usually occurs during adolescence from age 12 to 20. The major tasks during this stage are to integrate tasks of earlier stages (developing trust, self-control, sense of purpose, and self-confidence) and to develop a strong sense of the independent self.

A patient who looks at a picture of red roses and perceives a monster dripping blood from its mouth is experiencing: An illusion A delusion A hallucination Magical thinking

Illusion Rationale: Patient misinterprets real external stimuli, such as perceiving the picture of the red roses as a monster with blood dripping from its mouth. Delusion: Patient has false personal beliefs despite evidence to the contrary, such as a somatic delusion in which the patient has false ideas about the functioning of his/her body. Hallucination: Patient experiences false sensory perceptions (auditory, visual, tactile, gustatory, and olfactory). Magical thinking: Patient believes that thoughts have power to control others, situations, or things.

The primary focus of the Substance Abuse and Mental Health Services Administration (SAMHSA) is to: Reduce the costs associated with substance abuse and mental health. Make information, services, and research about substance abuse and mental health more easily accessible. Provide continuing education courses regarding substance abuse and mental health issues to healthcare providers. Serve as a political action committee to promote improvements in care for those with substance abuse or mental health issues.

Make information, services, and research about substance abuse and mental health more easily accessible. Rationale: The primary focus of the Substance Abuse and Mental Health Services Administration (SAMHSA), an agency within the U.S. Department of Health and Human Services, is to make information, services, and research about substance abuse and mental health more easily accessible and to reduce the impact of these issues on communities. SAMHSA has a number of Strategic Initiatives, such as "Trauma and Justice," and "Prevention of Substance Abuse and Mental Illness," as well as advisory councils and committees.

If a patient is being evaluated for depression and laboratory results show that the patient's free T4 level is 0.6 ng/dL (normal value 0.8 to 1.5 ng/dL) and the TSH level is 7.4 U/mL (normal value is 0.4 to 4.0 mIU/L), this suggests that depression: May result from hypoparathyroidism related to pituitary dysfunction May result from hypothyroidism related to thyroid dysfunction May result from hyperparathyroidism related to thyroid dysfunction Is likely unrelated to thyroid dysfunction

May result from hypothyroidism related to thyroid dysfunction Rationale: If a patient is being evaluated for depression and laboratory results show that the patient's free T4 level is 0.6 ng/dL (normal value 0.8 to 1.5 ng/dL) and the TSH level is 7.4 mIU/mL (normal value 0.4 to 4.0 mIU/L), this suggests that depression may result from hypoparathyroidism related to thyroid dysfunction. Typically, the TSH level rises to stimulate the thyroid to produce T4, so the levels may remain normal for a while because of the increased TSH or may begin to fall. If thyroid dysfunction were related to pituitary dysfunction, the TSH level would generally be decreased instead of elevated.

When utilizing sensory stimulation therapy (SST) to improve cognition in a patient with dementia, it is essential to choose sensory input that is: Meaningful to the patient Identified through testing Easy to demonstrate Beneficial to multiple patients

Meaningful to the patient Rationale: If utilizing sensory stimulation therapy (SST) to improve cognition in a patient with dementia, it is important to choose sensory input that is meaningful to the patient. For example, the psychiatric and mental health nurse may show the patient family pictures and talk with the patient about the family, encouraging the patient to retrieve memories, or may play music that the patient has previously enjoyed. Certain smells, such as perfume or food smells, may also be used to elicit memories.

A patient has undergone electroconvulsive therapy (ECT) and is awakening from the anesthesia. The initial patient response that is most expected is: Sedation Memory impairment Agitation Anger

Memory impairment

If a patient with substance abuse disorder states he has been using "beanies," the psychiatric and mental health nurse should understand that the patient is referring to: Marijuana Cocaine Methamphetamine Heroin

Methamphetamine Rationale: If a patient with substance abuse disorder states he has been using "beanies," the psychiatric and mental health nurse should understand that the patient is referring to methamphetamine, which is also sometimes referred to as "blue devils," "crank," and "crystal" as well as any number of local names. Marijuana is commonly called "weed," "Aunt Mary," "Mary Jane," and "pot." Cocaine may be called many names, including "coke," "blow,", "snow," and "sugar." Heroin may be called "horse," "H," "Aunt Hazel," "smack," and "charley" as well as many less commonly used names.

According to cognitive behavioral therapy (CBT), the type of automatic thought exemplified when a patient states, "My mother thinks I'm a failure," is: Personalizing All-or-nothing Discounting positives Mind reading

Mind Reading Rationale: According to cognitive behavioral therapy (CBT), the type of automatic thought exemplified when a patient states, "My mother thinks I'm a failure," is mind reading because the patient is assuming to know what is in another person's mind (although this would not hold true if the mother actually stated that the patient was a failure). An example of discounting positives is, "Of course I passed the test. The teacher made it too easy." All-or-nothing leaves no room for another interpretation: "Everyone knows I'm stupid." Personalizing brings everything back to the individual, "He's successful because of my advice and help."

A patient with autism spectrum disorder level 1 cannot judge the intention behind commands, often becoming distraught over simple directions, such as "eat your lunch now," or ignoring important directions, such as "leave by the fire exit." The term for this type of deficit is: Impaired social interaction Mind blindness Meltdown Stereotypy

Mind blindness Rationale: If a patient with autism spectrum disorder level 1 cannot judge the intention behind commands, becomes distraught over simple directions ("eat your lunch now"), and ignores important directions ("leave by the fire exit"), the term for this type of deficit is mind blindness. This same deficit interferes with patients' abilities to recognize faces. Mind blindness may contribute to impaired social interaction. A meltdown may begin with a tantrum but is more intense as the patient totally loses control and may endanger self or others. Stereotypy is rigid obsessive behavior. These deficits all result in impaired social interaction.

According to CBT principles, if a patient is unable to appreciate positive accomplishments but focuses on negative aspects, the automatic thought or cognitive error that the patient is using is: Dichotomization Magnification Catastrophizing Minimization

Minimization Rationale: According to CBT principles, if a patient is unable to appreciate positive accomplishments and focuses on negative aspects, the automatic thought or cognitive error that the patient is using is minimization. Other automatic thoughts include overgeneralization, mental filtering, catastrophizing, magnification, dichotomization, arbitrary inference/fortune telling, and personalization. CBT theorizes that one's perceptions influence behavior and emotions. Patients learn to identify automatic thoughts and then to see how those thoughts affect their feelings.

If a patient complains of difficulty focusing attention on more than an immediate task and difficulty concentrating as well as experiencing frequent headaches, GI upset, and muscle tension, the level of anxiety would likely be classified as: Mild Moderate Severe Panic

Moderate Rationale: If a patient complains of difficulty focusing attention on more than an immediate task and difficulty concentrating as well as experiencing frequent, headache, GI upset, and muscle tension, the level of anxiety would likely be classified as moderate. Patients often function better with mild anxiety but may feel restless and complain of insomnia and hypersensitivity to noise or other distraction. With severe anxiety, patients may have difficulty completing tasks or solving problems and behavior may focus on relieving anxiety. Physical symptoms may resemble a panic attack. With panic, patients cannot think or act rationally and may experience hallucinations and delusions.

The anxiety level at which a patient is typically alert, energized, and effective at problem solving because of the ability to focus narrowly is: Mild Moderate Severe Panic

Moderate Rationale: The anxiety level at which a patient is typically alert and energized and effective at problem solving because of the ability to focus narrowly is moderate. Some anxiety is common, for example, when a person has a problem to be solved or a job to complete. With mild anxiety, the person may experience random thought processes but can carry out habitual activities without difficulty. With severe anxiety, problem solving becomes difficult and the patient may experience physical signs of anxiety (tachycardia, sweating, diarrhea). With panic, the person is unable to function effectively and dissociation may occur.

Cattell-Horn-Carroll (CHC) Theory

Most recent that incorporates g, gf, gc, and specialized abilities into a hierarchical model of cognitive abilities and intelligence

A 30-year-old woman with a history of bulimia nervosa (BN) since adolescence is receiving therapy to help her want to change, accept responsibility for change, and remain committed to change. The patient is guided through different stages, beginning with precontemplation. This type of therapy is referred to as: Cognitive behavioral therapy (CBT) Motivation enhancement therapy Interpersonal psychotherapy Family therapy

Motivation enhancement therapy Rationale: Antidepressant Fluoxetine (Prozac®) is FDA-approved for BN. CBT-BN 1) Psychoeducation and strategies to eat normally and avoid binging and purging. 2) Food choices expand and dysfunctional attitudes, beliefs, and avoidance behaviors are identified. 3) Maintenance and relapse- prevention strategies are covered. Interpersonal psychotherapy 1) Interpersonal context of disorder are analyzed and problem areas identified. 2) Focus on problem areas. 3) Progress is monitored, but client is not advised to pay attention to patterns of eating or body attitudes. Family therapy Family assumes responsibility for ensuring the client eats a nutritious and adequate diet. Family conflicts are explored.

If a patient's family caregivers are interested in taking classes or training to better help them assist the patient and to cope more effectively with the patient's illness, the most appropriate referral is to the: National Alliance on Mental Illness (NAMI) Substance Abuse and Mental Health Services Administration (SAMHSA) American Psychiatric Nurses Association (APNA) National Institute of Mental Health (NIMH)

National Alliance on Mental Illness (NAMI) Rationale: If a patient's family caregivers are interested in taking classes or training to better help them assist the patient and cope more effectively with the patient's illness, the most appropriate referral is to the National Alliance on Mental Illness (NAMI). . Family-to-FamNAMI's Family-to-Family program is especially intended for family caregivers of those with severe mental illnessily comprises a 12-week course that is free. NAMI Basics is a course intended for parents/caregivers of children and adolescents with mental illness.

Which of the following provides unit level data regarding psychiatric physical and sexual assaults to help determine the quality of nursing care? National Center for Health Statistics (NCHS) National Database of Nursing Quality Indicators (NDNQI) National Death Index (NDI) MedlinePlus

National Database of Nursing Quality Indicators (NDNQI) Rationale: NDNQI provides unit level data to help determine the quality of nursing care by comparing data with other units and national averages. The NCHS compiles all types of health information from multiple sources in order to help document health status, monitor trends, support research, and identify needs and disparities. The NDI has provided records of death since 1979, compiled from state offices of vital statistics. This information may be used by researchers for statistical purposes only. MedlinePlus is a website produced by the National Library of Medicine to provide the general public access to information about health.

When assessing the patient's level of consciousness for a mental status exam, the psychiatric and mental health nurse notes that the patient is slow to respond and appears to have a decreased level of awareness, ignoring mild to moderate stimuli (such as the sounding of a call bell and the yelling of another patient). This level of consciousness may be described as: Clouded consciousness Delirium Obtundation Stupor

Obtundation Rationale: If, when assessing the patient's level of consciousness for a mental status exam, the psychiatric and mental health nurse notes that the patient is slow to respond and appears to have a decreased level of awareness, ignoring mild to moderate stimuli (such as the sounding of a call bell and the yelling of another patient), this level of consciousness may be described as obtundation. Levels of consciousness may include normal, clouded consciousness (inattention), delirium (abnormal state with fluctuating symptoms), stupor (responds only to strong stimuli), and coma (nonresponsive).

When assessing a 35-year-old Arab American female, the psychiatric and mental health nurse notes that, while discussing her family, the patient uses a louder voice than while discussing other issues. This probably means that issues about her family are: A private matter A cause for shame Of lesser importance than other issues Of special importance

Of Special Importance Rationale: If, when assessing a 35-year-old Arab American female, the psychiatric and mental health nurse notes that, while discussing her family, the patient uses a louder voice than while discussing other issues, this probably means that issues about her family are of special importance because speaking more loudly about important issues is characteristic of Arab Americans. People in this culture often stand close to others but avoid physical and eye contact with those of the opposite gender. However, it's important to remember what holds true in general for a culture may not hold true for an individual in the culture.

If a psychiatric and mental health nurse with many years of experience observes that a new nurse lacks essential skills, the most productive approach is to: Suggest the nurse take some continuing education courses. Provide study materials to help improve the nurse's skills. Report the nurse's lack of skills to the department head. Offer to serve as a mentor for the nurse.

Offer to serve as a mentor for the nurse. Rationale: If a psychiatric and mental health nurse with many years of experience observes that a new nurse lacks essential skills, the most productive approach is to offer to serve as a mentor for the nurse. Many new nurses lack essential skills because they have little experience to draw from and may be overwhelmed with the responsibilities of working. Mentoring is usually an ongoing process that lasts for months and even a year or more. Mentoring may be a formal or informal arrangement.

If a violent adult patient requires physical restraints, the patient be must be evaluated by a licensed independent practitioner within: 30 minutes One hour Two hours Four hours

One hour Rationale: If a violent adult patient requires physical restraints, the patient must be evaluated by a licensed independent practitioner within one hour of having the restraints applied. The same applies for the use of seclusion. Additionally, the patient must be evaluated personally every 8 hours after the initial evaluation, and physician's orders are required every 4 hours to continue the restraints or isolation. A nurse must closely supervise the patient and document an assessment every one to two hours.

Which of the following is an appropriate intervention for a nursing diagnosis of "disturbed thought processes?" Encourage patient to discuss delusions. Give detailed explanations about unit procedures. Keep a dim light on during the night to comfort the patient. Orient the patient to reality frequently and in various ways.

Orient the patient to reality frequently and in various ways. Rationale: The appropriate intervention for a nursing diagnosis of "disturbed thought processes" is to orient the patient to reality frequently and in various ways, such as by placing clocks within view and large signs as reminders. Explanations should be kept simple to avoid overloading the patient, and the psychiatric and mental health nurse should speak slowly and in a quiet voice to avoid agitating the patient. The patient should not be encouraged to discuss the delusions but should be encouraged to discuss real events or people.

The evidence-based Suicide Assessment Five-step Evaluation and Triage (SAFE-T) tool indicates that a patient has modifiable risk factors for suicide and strong protective factors, resulting in an overall low-risk factor although the patient admits to thoughts of death but denies a plan or intent. The intervention that is most indicated is: Outpatient treatment and crisis numbers Crisis plan and crisis numbers Admission to inpatient facility and crisis plan Admission to inpatient facility with suicide precautions

Outpatient treatment and crisis numbers Rationale: Since the evidence-based SAFE-T tool indicates that the patient is at low risk for suicide because risks (such as access to guns and health concerns) are modifiable and protective factors (such as religious beliefs and social supports) are strong, the intervention that is most indicated is outpatient treatment with crisis numbers to call if the patient needs support. The SAFE-T tool has 5 steps: (1) assessment of risk factors, (2) assessment of protective factors, (3) suicide inquiry (specific questions about plans, intent, ideation), (4) assignment of risk level (low, moderate, high) and appropriate intervention, and (5) documentation and plans.

A patient who is morbidly obese (>400 pounds) has frequently binged on food, eating until she vomits. Which community resource or support group is likely to be most helpful? Weight Watchers® Home meal delivery (Meals-on-Wheels) Overeaters Anonymous® Recovery International®

Overeaters Anonymous® Rationale: Overeaters Anonymous® is a 12-step support group that is focused on helping patients recover from compulsive eating as well as any eating disorder, such as anorexia or bulimia. Because participants often have severe weight problems or eating disorders, a patient who is morbidly obese may feel more comfortable in this group rather than Weight Watchers®, which attracts a different range of participants. Overeaters Anonymous® addresses not only the physical problems associated with compulsive eating but also emotional aspects. It is a peer-led group.

A patient with phobic disorder has a nursing diagnosis of social isolation. The most appropriate outcome is that the patient will: Be able to function despite presence of phobic object. Participate in group activities voluntarily. Carry out role-related activities. Acknowledge the need for social connections.

Participate in group activities voluntarily. Rationale: An appropriate outcome for a patient with a phobic disorder and a nursing diagnosis of social isolation is "Patient will participate in group activities voluntarily," as this action would demonstrate that the patient no longer feels compelled to be isolated. The psychiatric and mental health nurse should remain supportive and honest, attending meetings with the patient if necessary to alleviate fears, teaching the patient thought stopping activities to alleviate anxiety, and providing positive reinforcement.

The basic concept of the recovery model for mental health is: Patient compliance Patient education Patient advocacy Patient empowerment

Patient Empowerment Rationale: The basic concept of the recovery model for mental health is patient empowerment. Treatment is patient directed, and the philosophy is that recovery is possible. The recovery model views the person holistically and uses SAMHSA's four dimensions of support: health (physical and mental), home (safe, stable), purpose (meaningful activities/routines), and community (social relationships). The support system (friends, family, community services) for the patient is especially important to recovery.

f a patient with substance abuse disorder has a nursing diagnosis of ineffective coping in the nursing care plan, an appropriate expected outcome would be: Patient makes positive choices. Patient begins to recognize maladaptive behaviors. Patient does not act on impulses. Patient verbalizes feelings.

Patient begins to recognize maladaptive behaviors.

Which of the following is a Serious Reportable Event (SRE) related to patient protection? Patient is raped by a member of the staff on the hospital grounds. Patient receives an electric shock from faulty wiring. Patient dies because of a medical error. Patient cuts his wrists while hospitalized.

Patient cuts his wrists while hospitalized. Rationale: The National Quality Forum's (NQF's) Serious Reportable Events (SREs) are those events that are harmful to patients. The SREs are divided into different areas of focus. Those events that focus on Patient Protection are especially applicable to psychiatric and mental health nursing. These events include (1) discharge of a patient unable to make decisions to other than an authorized person, (2) death or serious injury related to elopement/disappearance, and (3) suicide, attempted suicide, or self-harm resulting in serious injury while hospitalized.

Which of the following is an example of the ego defense mechanism of rationalization? Patient states she beats her child because the child needs to learn to have self-control. Patient who is prejudiced against other races accuses others in the group of being bigots. Patient attends outpatient therapy to placate spouse but refuses to participate. Patient who experienced loss of a child refuses to think about or discuss the child's death.

Patient states she beats her child because the child needs to learn to have self-control. Rationale: An example of the ego defense mechanism of rationalization is when a patient states that she beats her child because the child needs to learn to have self-control. The patient is trying to blame her bad behavior on the child so that she can avoid feeling guilty or acknowledging responsibility for her own behavior. Patients often try to present the rationalization in such a way that the behavior appears positive, such as by helping the child to achieve better self-control, rather than negative.

When conducting a community assessment for preventive measures as part of community-based care, the three primary factors to consider are: People, environment, and social systems People, costs, and time Time, environment, and costs Social systems, needs, and costs

People, environment, and social systems Rationale: When conducting a community assessment for preventive health measures as part of community-based care, the three primary factors to consider are: People: This includes family, friends, and community members who may help support or interfere with preventive health measures. Environment: This includes an assessment of the location and its surroundings as well as the domicile to determine if they are likely to support preventive health measures. Social systems: This includes assessing which community resources are available and how they may affect preventive health measures.

If a patient with schizophrenia persistently repeats the same ideas over and over, this pattern of speech would be documented as: Perseveration Clang association Incoherence Looseness of association

Perseveration Rationale: If a patient with schizophrenia persistently repeats the same ideas over and over, this pattern of speech would be documented as perseveration. With clang association, the person uses words that are related by sound or rhyming. Incoherence is uttering nonsensical words or statements. Looseness of association refers to the use of statements and topics that are unconnected.

A patient repeats the same word over and over, "cake," when asked if he'd like dessert. This is an example of: Associative looseness Perseveration Tangentiality Echolalia

Perseveration Rationale: Perseveration: Patient repeats the same word phrase over and over, such as "cake," in response to a question. Associative looseness: Patient shifts comments from one topic to another but is unaware the ideas may be incoherent. Tangentiality: Patient introduces unrelated topics and never returns to the point of the communication. Echolalia: Patient repeats words or phrases that the patient hears. For example, if a psychiatric and mental health nurse states, "It's time to go to bed," the patient might repeat "it's time to go to bed" over and over or may shorten it to "bed, bed, bed, bed."

According to the four phases of alcoholic drinking behavior (Jellinek), blackouts are a characteristic of: Phase I, Pre-alcoholic Phase II, Early Alcoholic Phase III, Crucial Phase IV, Chronic

Phase II, Early Alcoholic Rationale: According to the four phases of alcoholic drinking behavior (Jellinek), blackouts are a characteristic of Phase II, Early Alcoholic. Phases: Phase I, Pre-alcoholic: Using alcohol to relieve stress (often learned behavior from childhood). Phase II, Early Alcoholic: Sneaking drinks, experiencing blackouts, and reacting defensively about drinking. Phase III, Crucial: --Binge drinking, --losing control to physiological dependence, --displaying anger and aggression, and --showing a willingness to sacrifice almost everything for alcohol. Phase IV, Chronic: --Disintegrating physically and emotionally, --being intoxicated most of the time, --experiencing life-threatening adverse effects.

If a 16-year-old female is severely anorexic, weighing 85 pounds and experiencing amenorrhea, hair loss, and cardiac abnormalities, according to Maslow's Hierarchy of Needs, which of the following needs is most dominant in this patient? Physiological Safety and security Belonging/Love Self-actualization

Physiological Rationale: If a 16-year-old female is severely anorexic, weighing 85 pounds and experiencing amenorrhea, hair loss, and cardiac abnormalities, according to Maslow's Hierarchy of Needs, the need that dominates is physiological because the patient is literally starving herself to death. Physiological needs form the base of Maslow's hierarchy because these needs must be met first. The next level is safety and security needs followed by belonging and love needs, and esteem needs. The highest level is self-actualization.

If a psychiatric and mental health nurse is physically attacked by a patient and the nurse suffers a broken arm as a result, this would be a Serious Reportable Event (SRE) in the category of: Care management event Patient protection event Environmental event Potential criminal event

Potential criminal event Rationale: If a psychiatric and mental health nurse is physically attacked by a patient and the nurse suffers a broken arm as a result, this would be a Serious Reportable Event (SRE) in the category of potential criminal event. It is "potential" because it may not be criminal if the patient were delusional, for example. Other potentially criminal events include care ordered or provided by someone impersonating a healthcare worker, abduction of a patient or resident, and sexual assault of a patient or staff member.

Which of the following is an example of a situational crisis? Retirement Marriage Poverty Parenthood

Poverty Rationale: Poverty is an example of a situational crisis, which is acute response to a stressor that relates to external circumstances. Other situational crises may include losing a job, environmental conditions (storms, tornados, hurricanes), and trauma (auto accident, falls). A maturational crisis, on the other hand, are experiences that are associated with different stages of growth and development, including adolescences, marriage, empty-nest situation, and retirement.

During which stage of the therapeutic relationship does the nurse examine personal feelings about working with a patient? Pre-interaction Orientation Working Termination

Pre-interaction Rationale: The therapeutic relationship: Pre-interaction: The nurse obtains information about the patient from various sources and examines personal feelings about working with the patient. Orientation: The patient and nurse get to know each other, establish rapport, and develop a plan of action based on the patient's strengths and weaknesses and nursing diagnoses. Working: The nurse helps the patient engage in problem solving activities and to overcome resistant behaviors, evaluating progress toward goals. Termination: The nurse and patient should explore feelings about termination and plan for continuing care.

Which of the following tests is most accurate for determining acute changes in nutritional status to monitor dietary compliance for a patient with anorexia? Transferrin Total protein Albumin Prealbumin

Prealbumin Rationale: Prealbumin (transthyretin) is most commonly monitored for acute changes in nutritional status because it has a half-life of only 2 to 3 days. Mild deficiency: 10-15mg/dL Moderate deficiency: 5-9 mg/dL Severe deficiency: <5 mg/dL Prealbumin is a good measurement because it quickly decreases when nutrition is inadequate and rises quickly in response to increased protein intake. Protein intake must be adequate to maintain levels of prealbumin. Total protein and transferrin levels can be influenced by many factors. Albumin has a half-life of 18 to 20 days, so it is sensitive to long-term protein deficiencies more than short-term.

The public health model (Caplan) of mental health care is based on the concepts of: Primary, secondary, and tertiary prevention Education, research, and application Patient, family, and community Community care, independence, monitoring

Primary, secondary, and tertiary prevention Rationale: The public health model (Caplan) of mental health care is based on the concepts of primary, secondary, and tertiary prevention. Primary prevention focuses on both preventive efforts for the individual and the environment to assist people to increase their ability to cope and to decrease stressors in the community. Secondary prevention involves promptly providing effective treatment for identified problems. Tertiary prevention aims to prevent complications of existing conditions and to promote rehabilitation.

The patient's medication list includes both a monoamine oxidase (MAO) inhibitor (isocarboxazid), which the patient has taken for many years, and an SSRI (fluoxetine), which was recently prescribed by another doctor. The psychiatric and mental health nurse should advise the patient that this combination may result in: Neuroleptic malignant syndrome Hypotension Hypertensive crisis Serotonin syndrome

Serotonin Syndrome Rationale: If the patient's medication list includes both a monoamine oxidase (MAO) inhibitor (isocarboxazid) and an SSRI (fluoxetine), the psychiatric and mental health nurse should advise the patient that this combination may result in serotonin syndrome, which can be life- threatening. Symptoms include confusion, hallucinations, fever, and myopathy. MAO inhibitors are no longer in common use because of multiple food and drug interactions that increase risk to patients. Patients prescribed an MAOI should always be advised to notify the prescribing physician before taking any other medication or herbal product.

A patient at an outpatient clinic who had been taking an SSRI for depression reports having recently stopped the medication because the patient no longer felt symptoms of depression. However, during the visit, the patient complains of severe headache, nausea, diarrhea, muscle aches and twitching, and increased anxiety. The patient's temperature is elevated. The most likely cause of these symptoms is: Serotonin withdrawal Exacerbation of depression Anticholinergic delirium Tardive dyskinesia

Serotonin withdrawal Rationale: If a patient recently stopped taking an SSRI and complains of severe headache, nausea, diarrhea, muscle aches and twitching, increased anxiety, and elevated temperature, the most likely cause of these symptoms is serotonin withdrawal, which can be life-threatening. The body reacts to the sudden decrease in serotonin, so it's important to slowly decrease the dose of SSRIs over the course of two to four weeks or even longer. Patients taking SSRIs should be apprised of the danger of suddenly stopping the medication.

A patient who is being admitted to an outpatient program for recovering cocaine abusers states that he has a history of gonorrhea and chlamydia, has multiple sex partners, and uses condoms erratically, especially if he has been drinking. The most appropriate initial screening for the patient is for: Sexual addiction Hepatitis Sexually-transmitted diseases Alcohol abuse

Sexually-transmitted diseases Rationale: While the patient should not be imbibing alcohol at all when in a recovery program for cocaine abuse, the patient is participating in sexual activity that places him at high risk for sexually- transmitted diseases; the initial screening should be for STDs so that necessary treatment can be provided if the patient tests positive. Drug and alcohol abuse are often accompanied by other high-risk activities because of lack of inhibition, poor judgment, and low sense of self-esteem.

A 45-year-old male is married with two children. He lost his job six months ago and cannot make house payments. He has become increasingly temperamental, has explosive outbursts of anger, and attacks his wife during an argument, hitting her in the face and throwing her against the wall. The type of emotional crisis he is likely experiencing is: Adventitious/social Situational/dispositional Maturational/developmental Psychopathological

Situational/dispositional Rationale: Situational/Dispositional crises result from a response to an external stressor, such as the loss of a job, that leaves the person feeling helpless and unable to cope. Adventitious/Social crises result from natural disasters, violent crimes, acts of terrorism, or socially disruptive acts, such as rioting, over which a person has little control. Maturational/Developmental crises occur during major life transitions, such as getting married, leaving home, or having a child. Psychopathological crises result from a preexisting psychiatric disorder, such as schizophrenia.

According to Maslow's Hierarchy of Needs, which of the following nursing diagnoses would have priority? Risk for injury Ineffective coping Sleep deprivation Social isolation

Sleep deprivation Rationale: According to Maslow's Hierarchy of Needs, the nursing diagnoses would be prioritized in the following manner (first to last): Physiological needs: Sleep deprivation. Safety and security needs: Risk for injury. Love and belonging: Social isolation. Esteem (self and from others): Ineffective coping. The last need is for self-actualization, but Maslow's Hierarchy of Needs is predicated on the idea that one must meet the needs at one level before progressing to the next level; so many people are never able to meet the needs associated with self-actualization.

A patient with Alzheimer's disease is no longer able to communicate verbally. When advising other staff members on communication strategies, the psychiatric and mental health nurse should advise them to: Avoid verbal communication with the patient. Talk to the patient as though the patient can completely comprehend. Communicate with very simple words and statements only. Slow down their speech and use nonverbal body language to communicate.

Slow down their speech and use nonverbal body language to communicate. Rationale: If a patient with Alzheimer's disease is no longer able to communicate verbally, when advising other staff members on communication strategies, the psychiatric and mental health nurse should advise the staff members to slow down their speech somewhat and use nonverbal body language to communicate. This can include smiling, pointing, and touching. The degree of comprehension may vary, and patients may become easily agitated, so patience is critical. Nurses should make eye contact with the patient before attempting to communicate in order to avoid startling the patient.

Which of the following divisions of the International Society of Psychiatric-Mental Health Nurses (ISPN) actively promotes the autonomy of the advanced practice nurse? Society for Education and Research in Psychiatric-Mental Health Nursing (SERPN) Association of Child and Adolescent Psychiatric Nurses (ACAPN) International Society of Psychiatric Consultation-Liaison Nurses (ISPCLN) Adult and Geropsychiatric-Mental Health Nurses (AGPN)

Society for Education and Research in Psychiatric-Mental Health Nursing (SERPN) Rationale: The Internal Society of Psychiatric-Mental Health Nurses (ISPN) comprises four divisions, which were originally independent organizations but came together to form the ISPN. The division that actively promotes the autonomy of advance practice nurses is the Society for Education and Research in Psychiatric-Mental Health Nursing (SERPN). Since the organization's original inception in 1983 (as the Council of Dean and Directors of Graduate Programs in Psychiatric- Mental Health Nursing), the organization has focused on graduate education in the field and evidence-based practice.

What is the antidote for Tricyclic Antidepressants?

Sodium Bicarbonate

A prevention strategy that encourages physicians, nurses, and other healthcare providers to discuss substance abuse with all adolescents is an example of: Secondary prevention Universal primary prevention Indicated primary prevention Targeted primary prevention

Targeted primary prevention Rationale: This is an example of targeted primary prevention. Primary prevention strategies include: Targeted: Aimed at a select group or subgroup with perceived risk. Strategies may include encouraging physicians to intervene with brief advice, such as advising all adolescents about dangers of substance abuse. Universal: Aimed at the entire population; non-specific. These strategies may include mass marketing procedures, such as multimedia anti-drug campaigns aimed at the general public. Indicated: Aimed at individuals at high risk, such as adolescents in environments with heavy drug use. Secondary prevention includes efforts to prevent further drug abuse, such as Narcotics Anonymous.

If a psychiatric and mental health nurse is giving a series of classes about psychotropic drugs and symptom management to a group of patients with bipolar disease, this type of group would be classified as: Teaching Supportive therapy Self-help Task

Teaching Rationale: If a psychiatric and mental health nurse is giving a series of classes about psychotropic drugs and symptom management to a group of patients with bipolar disease, this type of group would be classified as teaching because the primary focus is on transmission of information rather than therapy, self-help measures, or specific tasks. Teaching groups usually are not open-ended but have a set number of classes at prescribed times. Teaching groups should include time for questions and answers and interactions among group members to facilitate recall.

Which of the following is a boundary violation on the part of the psychiatric and mental health nurse? Accepting a box of candy to be shared by the staff Finding a patient attractive Holding the hand of a frightened patient Telling a patient about breaking up with a fiancé

Telling a patient about breaking up with a fiancé Rationale: Sharing personal information, such as by telling about breaking up with a fiancé, is almost always a boundary violation unless it serves a real therapeutic purpose. Even then, self-disclosure should be done judiciously. Finding a patient attractive is not a concern unless the nurse acts on the attraction. Touch is a sensitive issue and can be easily misconstrued, but holding a patient's hand or touching an arm to comfort a patient is usually acceptable. Gifts generally should not be accepted, although candy to be shared with the entire staff is usually an exception.

The theory that outlines nine personality parameters to explain how children respond to events and describes the difficult child, the child who is slow to warm up to new people and circumstances, and the child who is easy to manage and adaptable is the: Resilience theory Social learning theory (Bandura) Temperament theory (Chase and Thomas) Theory of moral development (Kohlberg)

Temperament theory (Chase and Thomas) Rationale: Chess and Thomas's temperament theory describes nine personality parameters to describe how children (≥4 weeks) respond to events. These personality traits explain the difficult child, the child who is slow to warm up to new people and circumstances, and the child who is easy to manage and adaptable. Resilience theory describes the ability of children to function in healthy ways despite adverse circumstances. Bandura's social learning theory proposes that children learn from interacting with adults and their peers and through modeling behavior. Kohlberg's theory of moral development outlines the progressive stages in which children develop a sense of morality.

In an administrative model of shared governance, the person representing the psychiatric unit is probably: The department head A team leader Any member of the nursing staff Any member of the staff

The department head Rationale: In an administrative model of shared governance, the person representing the psychiatric unit is probably the department head because this model depends on the leaders of the institution. These leaders may preside over smaller councils, but they alone are represented on the primary legislative council. Councilor models may have a large number of councils that have some governance over their members. For example, each unit may have a council that sets work hours. In the congressional model, all nursing staff (or all staff) may be members of councils with varying degrees of autonomy.

A psychiatric and mental health nurse feels sorry for a patient because his family won't support him. The nurse offers to visit the patient's family as well as purchase some items for him. This nurse is: Showing empathy Violating professional boundaries Building a strong therapeutic relationship Exhibiting negligence

Violating professional boundaries Rationale: If a psychiatric and mental health nurse feels sorry for a patient who states his family won't support him and offers to visit the family as well as purchase some items for him, the nurse is violating professional boundaries by becoming over-invested in the patient and attempting to solve his problems for him rather than helping him to do so. Additionally, the nurse is establishing a relationship in which the patient may have unrealistic expectations of what the nurse will do, and this can lead to conflict.

According to Peplau's Interpersonal Theory, a patient who is miserly, suspicious, and envious of others has likely failed to complete developmental tasks associated with the stage of: Infancy (learning to count on others) Toddlerhood (learning to delay satisfaction) Early childhood (identifying oneself) Late childhood (developing participatory skills)

Toddlerhood (learning to delay satisfaction) Rationale: During the stage of toddlerhood, the child should learn to delay gratification and feel satisfied when delaying self-gratification is pleasing to others. If patients have not completed tasks of toddlerhood, as adults they may use exploitive and manipulative behavior with others, exhibit envy and suspiciousness toward others, hoard, exhibit miserliness, exhibit inordinate neatness/punctuality, have difficulty relating to others, and alter personality characteristics to fit the situation. A patient who has not completed toddlerhood tasks requires complete acceptance in order to feel safe and secure.

A 62-year old-male with fragile X syndrome has been diagnosed with fragile X tremor-ataxia syndrome. The psychiatric and mental health nurse should expect the patient to exhibit: Tremor and ataxia only Tremor, ataxia, mood changes, paresis, dementia Tremor, ataxia, mood changes, cognitive decline, dementia Tremor, ataxia, mood changes, cognitive decline, paresis

Tremor, ataxia, mood changes, cognitive decline, dementia Rationale: If a patient is diagnosed with fragile X tremor-ataxia syndrome (FXTAS), the psychiatric and mental health nurse should expect the patient to exhibit intention tremors, ataxia, mood changes (anxiety, depression), cognitive decline, and dementia. This neurological decline associated with FXTAS occurs later in life and increases with age with 17% of those between 50 and 59 exhibiting symptoms and 74% of those over 80 years. Early symptoms include difficulty writing, using utensils, and frequent falls. FXTAS rarely affects females.

A patient with schizophrenia and a history of violent behavior in response to "voices" has been pacing about his room and suddenly begins shouting at the nurse, "Get away from me! Let me out of here!" Considering the 5-phase aggression cycle, the patient is most likely in the phase of: Crisis Recovery Triggering Escalation

Triggering Rationale: The patient is most likely in the phase of escalation. The 5-phase aggression cycle includes: Triggering: Patient appears restless, irritable, pacing, tense, and exhibits increased perspiration, loud voice, and angry demeanor. Escalation: Patient may begin yelling and swearing and making threatening gestures, exhibiting hostility and loss of self-control. Crisis: Patient loses complete control and may begin hitting, spitting, throwing items, kicking, and screaming. Recovery: Patient begins to relax physically and emotionally, lowering voice and acting more rationally. Post-crisis: Patient experiences remorse and may cry or become withdrawn.

Mental health patients should be included in shared decision making and give informed consent: If they appear competent to comprehend and understand implications Unless they have been declared incompetent by court proceedings If they cooperate with treatment and appear to pose no threat If they ask to participate in shared decision making

Unless they have been declared incompetent by court proceedings Rationale: Mental health patients should be included in shared decision making and give informed consent unless they have been declared incompetent by court proceedings. Mental health patients, the same as other patients, are presumed to be competent under existing laws and have the right to make decisions about the type of care they receive, even if their decision is contrary to the advice of physicians. In order to be declared incompetent, a patient generally has to pose a threat to self or to others.

The clinical signs associated with borderline personality disorder include: Grandiosity, lack of empathy, and a need for attention and admiration from others Unstable relationships, self-injury, impulsive/unpredictable behavior, and disordered self- image Submissive role in relationships, passivity, self-doubt Exaggerated sensitivity to rejection, social inhibitions, lack of self-confidence

Unstable relationships, self-injury, impulsive/unpredictable behavior, and disordered self- image Rationale: The clinical signs associated with borderline personality disorder include unstable relationships, self-injury, impulsive/unpredictable behavior, and disordered self-image. Patients often make recurrent suicide gestures and feel insecure and worthless. Behavior is often erratic, and patients are often unable to establish long-term relationships with others. They may exhibit impulsivity in a number of areas such as excessive spending, sex, substance abuse, reckless driving, and binge eating. Borderline personality disorder is more common in females than males.

A 16-year-old girl was recently diagnosed with anorexia nervosa with symptoms being evident for one year. The psychiatrist is using the Maudsley method of treatment for anorexia nervosa, which stresses family involvement. The responsibility of the family during phase I of treatment is to: Prepare all meals and criticize lack of compliance with eating Encourage the patient to become more independent in controlling eating Change the dynamics of family interactions and encourage autonomy Use strategies to encourage the patient to eat more to restore weight

Use strategies to encourage the patient to eat more to restore weight Rationale: The Maudsley method of treatment for anorexia nervosa is used for adolescents who have had symptoms of the disorder for fewer than three years. There are three phases to the treatment regimen: Phase I: The focus is on weight restoration, so the family should use strategies to encourage the patient to eat more and to restore weight, avoiding any criticism or blame and using positive reinforcement. Phase II: The focus is on encouraging the patient to become more independent in controlling eating. Phase III: The focus is on changing the dynamics of family interactions and encouraging autonomy in the patient.


संबंधित स्टडी सेट्स

ENLIGHTENMENT - enlightened Despots examples and achievments

View Set

Geologic Time- Time Scale, Geologic Time Scale

View Set

Vehicle and Road Safety/Natural Laws (8.1-8.2)

View Set

Importance of the Ocean / Deepest Place on Earth

View Set

Preguntas con 'Saber' y 'Conocer'

View Set

chemistry check in ch 3.2 and 3.3

View Set

Designing Randomized Evaluations

View Set